You are on page 1of 39

- CHAPTER 02

Electrostatic Potential

R
And Capacitance

SI
Chapter Analysis w.r.t. Lost 3 Year's Board Exams
The analysis given here gives you an analytical picture of this chapter and will help you to
identify the concepts of the chapter that are to befocussed more from exam point of view.
Number of Questions asked in last 3 years
IT
2015 2016 2017
Delhi All India Delhi All India Delhi All India
Very Short Answer (1 mark) lQ lQ lQ lQ
Short Type I Answer (2 marks) lQ
Short Type II Answer (3 marks) lQ lQ 1Q lQ 1 Q._ ~Q
H
v_ ..J

Long Answer (5 marks) IQ 91

Value Based Questions (4 marks)


, ...
• In 2015, in Delhi set, one numerical question of 2 marks and one numerical question of 3
marks based on Potential Difference were asked.
O

In All India set, only one numerical question based on Combination of Capacitor was
asked.
• In 2016, in Delhi set, one question of 1 mark based on Potential Difference, one
question of 3 marks based on Parallel Plate Capacitor were asked. In All India set, one
M

question of 1 marks based on Dipole Moment, one question of 3 marks based on


Combination of Capacitor and one numerical question of 5 marks based on Dielectric
and Capacitor were asked.
• In 2017, in Delhi set, one question of marks based on Electrostatic Shielding and one
question of 3 marks based on Electric Dipole were asked. In All India set, one question of
1 marks based on Polarity of the Capacitor and one question of 3 marks based on
Parallel Plate Capacitor were asked.
On the basis of above analysis, it can be said that from exam point of view Potential Difference,
Dielectric and Capacitor, Electric Dipole and Parallel Plate Capacitor are most important
concepts of the chapter.
[TOPIC 1] Electrostatic Potential and
Electrostatic Potential Energy
1.1Electrostatic Potential For the diagram given below, potential difference
between points A and B will be same for any path.
The electrostatic potential at any point in an
electric field is equal to the amount of work

R
done in bringing the unit positive test charge
A~B
without acceleration from infinity to that
point. 4
. . Work done (W)
Electrostatic potential, V = -----'---'- Electrostatic Potential

SI
Charge (qo)
Its SI unit is volt (V) and 1V = 1 rc' and its
due to a Point Charge
dimensional formula is [ML2T-3 A-I]. Electrostatic potential due to a point charge q at any point
It is a scalar quantity. P lying at a distance r from it is given by V = _1_.1
41tEo r
NOTE Electrostatic potential is a state dependent
function as electrostatic forces are conservative The potential at a point due to a positive charge is
forces. positive while due to negative charge, is negative.
IT
• When a positive charge is placed in an electric field,
Electrostatic Potential it experiences a force which drives it from points of
Difference higher potential to the points of lower potential. On
the other hand, a negative charge experiences a force
The electrostatic potential difference between
driving it from lower potential to higher.
t,"yopoints in an electric field is defined as the
amount o[, work done in moving a unit
H
positive test charge from one point to the Electrostatic Potential
other point against electrostatic force without due to Electric Dipole
any acceleration (i.e. the difference of
electrostatic potentials of the two points in • Electrostatic potential due to an electric dipole at any
point P inclined at an angle 3 whose position vector
O

the electric field).


is r w.r.t. mid-point of the dipole is given by
-V - WAB
VB A- V = _1_.p cos e
qo
41tEo r2
where, WAB is work done in taking charge qo p
from A to B against of electrostatic force.
M

Also, the line integral of electric field from


initial position A to final position B along any
path is termed as potential difference
9
between two points in an electric field, i.e.

VB - VA =- f: E·dl
-q
11+-,---
0
2a
P
----+II

Electrostatic potential at point P due to a short dipole


+q

NOTE As, work done on a test charge by the


electrostatic field due to any given charge ( a< < r j is gt
IS given
by V = -_.1 ~p.i-
configuration is independent of the path, hence 41tEo r:
potential difference is also same for any path.
where, e is the angle between rand p.
CHAPTER 2 : Electrostatic Potential And Capacitance 37

Important Results For a single charge q, the potential is given by


• The electrostatic potential on the perpendicular v=_I_.1l
bisector due to an electric dipole is zero. 41tEo r
• Electrostatic potential at any point P due to a The shape of equipotential surface due to
system of n point charges ql' qz,.··, qn whose (i) line charge is cylindrical
position vectors are r1 , r2, ... , rnrespectively, is (ii) Point charge is spherical
given by

R
V=_I_ i .sc: Field
41t£o i;llr-r;! lines
where, r is the position vector at point P w.r. t.
the origin.

SI
• Electrostatic potential due to a thin charged , - kquiPotential
spherical shell carrying charge q and radius R surfaces
respectively, at any point Plying (spherical in shape)

(i) inside the shell is V = _I_.fL, for r « R Different properties of equipotential surface are given
41tEo R below:
(i) Equipotential surfaces do not intersect each
(ii) on the surface of shell is V = _1_. fL,
41tEo R other as it gives two directions of electric
IT
for r= R field at intersecting point which is not
possible.
(iii) outside the shell is V = _1_.Il, for r » R
41tEo r (ii) Equipotential surfaces are closely spaced in
the region of strong electric field and vice-versa.
where, r is the distance of point P from the
centre of the shell. (iii) Electric field is always normal to
equipotential surface at every point of it and
H
• Graphical representation of variation of electric
directed from one equipotential surface 'ii.tfH,
potential due to a charged shell at a distance r
higher potential to the equipotential surface
from centre of shell is given below:
at lower potential.
V
(iv) Work done in moving a test charge from one
1 q
O

V=-·- point of equipotential surface to other is zero.


4m:o r

Relation Between Electric Field


and Potential Gradient
Relation between electric field and potential
M

gradient is given by
O~--.~:R---r--.-----
dV . oV oV oV
B=-- i.e. E =-- E =-- E =--
dr x ox' y oy' z oz
Variation of potential due to charged shell where, negative sign indicates that the direction of
with distance r from its centre electric field is from higher potential to lower
potential, i.e. in the direction of decreasing potential.
1.2 Equipotential Surface NOTE (i) Electric field is in the direction of which the
potential decreases steepest.
A surface which has same electrostatic potential
(ii) Its magnitude is given by the change in the
at every point on it, is known as equipotential
magnitude of potential per unit displacement
surface.
normal to the equipotential surface at the point.
38 o ehopterwise eSSE Solved Papers PHYSICS

Potential Energy in 2. A point charge + Q is placed at point 0


as shown in the figure. Is the potential
an External Field difference (VA - VB) positive, negative
(i) Potential Energy of a Single Charge Potential or zero?
energy of a single charge q at a point with +0·-------------- •..-----·
o A B
position vector r, in an external field is qV(r),
Deihl 2016, Foreign 2016, Deihl 2011
where V(r) is the potential at that point due to

R
external electric field E. 3. A charge q is moved from a point A
above a dipole of dipole moment p to a
(ii) Potential Energy of a System of Two
point Bbelow the dipole in equatorial
Charges
plane without acceleration. Find the
U = q1V(rj) + q2V(r2) + qlq2 work done in this process.

SI
41tEOr12 A,
,,
where, ql ,q2 = two point charges at position ,,
, ,
vectors r) and r2, respectively
V (r)) = potential at r) due to the external field - q .--------~---, ----4 +q
,,
V(r2) = potential at r2 due to the external field ,,
,,
1.3 Electrostatic Shielding B All Indio 2016
IT
• The process which involves the making of a region 4. Why are electric field lines
free from any electric field is known as electrostatic perpendicular at a point on an
shielding. equipotential surface of a conductor?
Conductor All India 2015C

5. Two point charges q and -2q are kept


H
d distance apart. Find the location of
point relative to charge q at which
potential due to this system of charges
is zero. All India 2014C
It happens due to the fact that no electric field exist 6. The figure shows the field lines of a
O

inside a charged hollow conductor. Potential inside positive charge. Is the work done by
a shell is constant. In this way, we can also the field is moving a small positive
conclude that the field inside the shell (hollow . charge from Q to P positive or
conductor) will be zero. negative?
M

PREVIOUS YEARS' Foreign 2014

EXAMINATION QUESTIONS 7. For any charge configuration,


equipotential surface through a point
TOPIC 1 is a normal to the electric field.
Justify.
o 1 Mark Questions Deihl 2014

1. Does the charge given to a metallic sphere 8. What is the geometrical shape of
equipotential surfaces due to a single
depend on whether it is hollow or solid? Give
isolated charge? Deihl 2013, All India 2010 C
reason for your answer. De!l!L2017
CHAPTER 2 : Electrostatic Potential And Capacitance 39

9. What is the amount of work done in


moving a point charge around a circular
o 2 Marks Questions
arc of radius r at the centre of which 22. Two point charges ql and q2 are located at
another point charge is located? rland r2' respectively in an external
All India 2013 C electric field E. Obtain the expression for
10. Two charges 21lC and -21lC are placed at the total work done in assembling this
points A and B, 5 em apart. Depict an configuration. Delhi 2014C
equipotential surface of the system.
23. Two closely spaced equipotential surfaces
Delhi 2013C

R
A and Bwith potentials V and V +OV;
11. Why electrostatic potential is constant (where OV is the change in V) are kept ol
throughout the volume of the conductor distance apart as shown in the figure.
and has the same value as on its surface? Deduce the relation between the electric
Delhi 2012 field and the potential gradient between

SI
12. Why the potential inside a hollow them. Write the two important
spherical charged conductor is constant conclusions concerning the relation
and has the same value as on its surface? between the electric field and electric
Foreign 2012 potential. Delhi 2014C
13. Why there is no work done in moving a B V+ sv
charge from one point to another on an -/--,V
equipotential surface? Foreign 2012
IT
14. A hollow metal sphere ofradius 5 em is
charged such that potential on its surface
is lOV. What is the potential at the centre
of the sphere? All India 2011 24. Calculate the amount of work done to
15. Can two equipotential surface intersect dissociate a system of three charges, two
each other? Justify your answer. Delhl2011C of lllC and one of -41lC placed on the
H
vertices of an equilateral triangle of side
16. Draw equipotential surfaces due to a 10 cm. All India 2013C
single point charge. All India 2011C
25. A test charge q is moved without
17. Name the physical quantity whose 81 unit acceleration from A to C along the path
is JC-1. Is it a scalar or a vector quantity? from A to B and then from B to C in
O

All India 2010 electric field E as shown in the figure.


18. What is the work done in moving a test (i) Calculate the potential difference
charge q through a distance of 1 em along between A and C.
the equatorial axis of an electric dipole? (ii) At which point (of the two) is the
All India 2009 electric potential more and why?
M

19. Define the term potential energy for


A ·,:.-----~-~D]--r;B
charge q at a distance r in an external "
.••...
..,:
field. 5 crri> :3 em
All Indio 2009 "
..... :
I

''''C E
20. The potential due to a dipole at any point
on its axial line is zero. Correct or Wrong?
All India 2012; Foreign 2009
All India 2009C
26. Draw a plot showing the variation of
21. What is the electric potential due to an
(i) electric field (E) and (ii) electric
electric dipole at an equatorial point?
potential (V) with distance r due to a
All India 2009
point charge Q. Delhi 2012
40 o Chopterwise CBSE Solved Papers PHYSICS

27. Two uniformly large parallel thin plates 33. (i) Write two characteristics of
having charge densities + cr and - cr are equipotential surfaces.
kept in the X Z- plane at a distance d (ii) Draw the equipotential surfaces due
apart. Sketch an equipotential surface to an electric dipole. All India 2009C
due to electric field between the plates. If
a particle of mass m and charge - q o 3 Marks Questions
remains stationary between the plates. 34. (i) Derive the expression for the electric

R
What is the magnitude and direction of potential due to an electric dipole at a
this field? Delhi 2011 point on its axial line.
28. A dipole with its charge -q and +q (ii) Depict the equipotential surfaces due
located at the points (O,-b, 0) and to an electric dipole. Delhi 2017
(O,+b,O)is present in a uniform electric 35. Define an equipotential surface. Draw

SI
field E. The equipotential surfaces of this equipotential surfaces
field are planes parallel to the YZ-plane. (i) in case of a single point charge
(i) What is the direction of the electric (ii) in a constant electric field in
field E? Z-direction. Why the equipotential
(ii) How much torque would the dipole surfaces about a single charge are not
experience in this field? Delhi 2010C equidistant?
(iii) Can electric field exist tangential to
29. Two point charges 31lC and -31lC are
an equipotential surface? Give reason.
placed at points A and B, 5 em apart.
IT
All India 2016
(i) Draw the equipotential surfaces of 36. (i) Depict the equipotential surfaces for a
the system. All India 2011C system of two identical positive point
(ii) Why do equipotential surfaces get charges placed a distance d apart.
close to each other near the point (ii) Deduce the expression for the potential
charge? energy of a system of two point charges
H
30. Find out the expression for the potential ql and q 2 brought from infinity to the
energy of a system of three charges ql ,q2 points with positions r1 and r2
respectively, in presence of external
and q3 located at r1 ' r2 and r3 with
electric field E. Delhi 2Dl0
respect to the common origin O. Delhi 2010C

o Explanations
O

31. Draw three equipotential surfaces


corresponding to a field that uniformly
increases in magnitude but remains 1. In case of metallic sphere, charge given to it is
mostly resides on its surface. Therefore, there is no
constant along Z-direction. How are
difference whether the sphere is hollow or solid. As
these surfaces different from that of a in both the cases, the charge that will reside will be
M

constant electric field along Z-direction? same. (1)


All India 2009
2. According to question,
32. (i) Can two equipotential surfaces
intersect each other? Give reasons.
+0.
o A
._uu .. B
---(1-
(ii) Two charges - q and + q are located • (2----+
at points A (0,0, - a) and B(O, 0, +a)
respectively. How much work is r2> r1 (given)
done in moving a test charge from Potential at point A due to charge + Q,(VA) = kQ
point P (7,0,0) to Q (- 3,0, O)? r1
Delhi 2009
Potential at point B due to charge + Q, (VB) = kQ
r2
(1I2J
CHAPTER 2 Electrostatic Potential And Capacitance 41

1 1 6. Work done by charge is given by


As VA OC - and VB OC - and '2>'] so, VA> VB
'] '2 W = q (potential at Q - potential at Pl.
Thus, (VA- VB) is positive. (1/2)
where, q = small positive charge (1/2)
3. According to questions,
The electric potential at a point distant, due to
A
,,, ,,, the field created by a positive charge Q is given by
,, ,,
v=_l_q_
r .: ',r
41tEo r
,," Y " ,,
,, ,,
'p <'Q => Vp > VQ

R
(1/2)
-<:;' x x '+q
So, work done will be negative.
B 7. No work is done in moving the test charge from
Total potential at point A due to + q charge, one point of an equipotential surface to the other.
(1/2)
VA = kq = kq

SI
r !. .. WB-WA=o=-fE.dl => E·dl=O
(x2 + i)2

Total potential at point A due to - q charge, Hence, ei dl (1/2)

V~ = - kq = - kq 8. The equipotential surfaces produced by a single


, !. point charge are spherically symmetrical charge
(x2 + i)2 which are concentric. In concentric spheres which
So, net potential = VA + V~ = 0 are as shown below in the figure, the lines of
force point radially outwards, so they are
Similarly at point B, potential will be O. perpendicular to the equipotential surfaces at all
IT
So, net work done = l1V x q = O. (1) points.
4. Electric field is always normal to the
equipotential surface at every point, because no
work is done, as W = qO(VA - VB) => VA - VB = 0
hence, W = 0.
If the field were not normal to the equipotential
H
surface, it would have a non-zero component
along the surface. So, to move a test charge
against this component, a work would have to be
done. But there is no potential difference between
any two points on an equipotential surface and
O

consequently no work is required to move a test


charge on the surface. Hence, the electric field
must be normal to the equipotential surface at
every point. (1)
Equipotentiol surfoces for 0 point charge
5. According to the question, q A = q and qB = -2q
. . .
M

A P B 9. The amount of work done in carrying a charge on


qA qB equipotential surface is always zero (potential
I~ d .1 difference is zero).
I+-X~
f+-a-x __ 10. Given, qA=2IlC=2xlO-6C
qB =-2IlC=-2xlO-6Cand ,=5cm
kqA kqB
VPA =-- => VPB =-- A +-x---;(5-X)-B
x (d-x)
I--- 5em-----.j
As, the potential due to system of charges is zero,
kq 2kq .. Potential.
hence VpA + VPB = 0 => - = --
x (d-x) 2XlO-6 -2XIO-6
V= +-------:;-
41tEoXxlO-2 41tEO(5-x) X 10-2
d-x=2x=> 3x=d=> x=~
3 (1)
42 o ehapterwise eBSE Solved Papers PHYSICS

2xlO-6 2xlO-6 18. The electric potential at any point on equatorial


[':V = 0]
41t£oxX lO-z 411:EO(5-x) xlO-z line due to an electric dipole is equal to zero.
x=5-x ~x=2.5cm m No work is done in moving a test charge along
equatorial line, as given below:
11. Since, electric field intensity inside the conductor
W = qL\V= q(VB - VA)
is zero. So, electrostatic potential is a constant.
= _ L\V VB = VA = 0 ~ W =0 m
But, E
L\r 19. The electric potential energy at any point lying at
= 0, =0 or Vz - VI = 0, V2 = a distance r from the source charge q is equal to

R
E L\V VI
The potential at every point inside the conductor the amount of work done in moving unit positive
remains same. (1l test charge from infinity to that point without
any acceleration against electrostatic force. (1l
12. Electric field inside the hollow spherical charged
conductor is zero. So, no work is done in moving 20. Wrong, as potential due to an electric dipole is
zero on equatorial line in spite of axial line.

SI
a charge inside the shell. Thus, potential is
constant and therefore, equal to its value at the The potential due to a dipole at any point on
surface, i.e. V = _1_. !L equatorial line is zero, not on axial line. (1l
411:~ R ru 21. Zero, as potential on equatorial point due to
13. An equipotential surface is a surface at which charges of electric dipole, is equal in magnitude
every point electric potential is same. but opposite in nature and hence, their resultant
As, work done in moving a charged particle from
huro. ru
one point to another is defined as 22. Work done in bringing the charge ql from infinity
IT
L\W = q(L\V) to position rl
On an equipotential surface, the potential WI = ql V(rd ... (i) (1/2)

remains constant. So, L\V = zero Work done in bringing charge q2 to the position rz
~ Work done, L\W = 0 (1l Wz = qzV(rz) +.3J.!J..L ... (ii) (1I2l
411:Eor12
14. Refer to Ans. 2 [Ans.lOV] rn
15. (i) No, two equipotential surfaces cannot Hence, total work done in assembling the two
H
intersect each other because two normals can charges.
be drawn at intersecting point on two surfaces W=WI + Wz
which give two directions of Eat the same point
From Eqs. (i) and (ii),we get
which is impossible. (112l
(ii) Also, two values of potential at the same point W =ql V(rd+ qzV(rz)+~
O

411:~r12 (1l
is not possible. (112l
23. Work done in moving a unit positive charge along
16. Equipotential surfaces due to a single point distance 8Z,
charge are concentric sphere having charge at the
centre. IEd8Z=VA-VB =V-(V+8V) =-OV E=-8V
8t (1)
M

(i) Electric field is in the direction in which the


,
,>,\x
\7 Equipotential
surfaces
potential decreases steepest.
(ii) Magnitude of electric field is given by the
(1/2)

change in the magnitude of potential per unit


6--1.-+--+-'--_ Line of displacement normal to the equipotential
Eforces surface at the point. (1/2)

24. Given, charges ql = lfJ.C qz = -1fJ.C and q3 = -4fJ.C


Each side of equilateral triangle, r = 10 em
- -_ ... Potential energy, U = Total work done to assemble
(1) the three charges U = WI + Wz + W3
6
17. rc' is the 51 unit of electric potential. It is a U = _1_[1 x 10-6(-4 X 10- ) +
scalar quantity. (1I2+1I2l 411:Eo 010
CHAPTER 2 : Electrostatic Potential And Capacitance 43

1 X10-6(l X10-6) + -4 X10-6(1X10-6)] Potential due to a point charge,


010 010 (1) V=_l_g ~Vo<~
1 ~C 41tEo r r
The variation of electrostatic potential with
10Cm/\10Cm distance, i.e. V 0< ~ and also the variation of
r
electrostatic field with distance, i.e. E 0< ~ •
+1~C~-4~C r
10cm

R
U = _1_ X10-12[-4xlO+ 1O-4xlOj Due to a single charge, F 0< ~ and E 0< ~ but
41t£o
r r
V 0< ~,where, r is the distance from the charge.
U = -9 X109 X10-12 x 70 ~ U = -0. 630J r m
Work done to dissociate the system of charges 27.
II J-Y X

SI
W = -v ':;.630J (1)
+0 / + + + + + + +
25. (I}',: blectric field intensity and potential difference Equipotential
are related as surface v-----;.v-~-O------/'d
E = _L\V {- 1 I Z
M
= -EM = -4E d/2r-- ---.
L\V ~ Vc - VA
.: By pythagoras theorem.ac? = AB2 + BC2]
(1)
-01/+ + + + + + + I
[ ~ AB2 = 52 - 32 ~ L\r=M ~L\r= 4
The equipotential surface is at a distance d/2 from
IT
(ii) As Vc - VA = 4E is negative either plate in XZ-plane. -q charge experiences a
force in a direction opposite to the direction of
.. Vc < VA
electric field.
Potential is greater at point A than point C, as
., - q charge balances when
potential derreases along the direction of
electric field. (1) qE=mg

26. Electric fit..J due to a point charge, . E= mg


H
1 Q 1 q
E=---,Eo<,
41tEo r2 r: The direction of electric field along vertically
downward direction. The XZ-plane is so chosen
A graph showing variation of electric field (E) that the direction of electric field due to two plates
and electric potential (V) with distance (r) are is along vertically downward direction, otherwise
O

shown below. (1) weight (mg) of charge particle could not be


VE balanced. (1)
5.0 r-r....,.---,-..,--.,.-.--.-,---,--.--,
Y
4.5
4.0 V =_1_ s.
4~ r Equipotential
:::. 3.5 1---- 1/r surface --
M

E=_1_ q --1/r2
1

GJ =i 3.0 4~ (i ----X
:2 ~
Q)-
2.5
~ ci: 2.0
n 1.5
z E (Vertically
downward)
(1)
&l 1.0
0.5 ------- _ 28. The direction of electric field is perpendicular to
O~~~~~~~~~~ the equipotential surface.
0.5 1.0 1.5 2.0 2.5 3.0 3.5 4.0 4.5 5.0
Distance (r) _ (i) The direction of electric field is along X-axis as
it should be perpendicular to equipotential
Variation of electrostatic potential V and electric surface lying in yz- plane.
field E with distance r Length of the dipole = 'lb
44 o ehapterwise eSSE Solved Papers PHYSICS

As dipole's axis is along the Y-axis. :. Net potential energy of the system,
:. Electric dipol~ moment, U = UI2 + U23 + U3I
~q(~j ru U = _1_ [3J..!!.L + .sas: + .s.ss: lJ
(ii) Electric field, E = E i 41tEo Ir2 - rl I [r 3 - r21 Ir 3 - rl I (1)
t=pxE=q(2b)JXEl
A A A 31. Electric field is always directed from higher to
= + 2qbE(j x i) =2qbE(-k) lower potential and perpendicular to the

R
Torque, I t I = 2qbE (1) equipotential surfaces.
Alternative method The figure is shown as below:
Z
E is directed along X-axis.
Dipole moment p = q(2b) from (0, =b, 0) to (O,b,O),
i.e. along Y-axis. (1)

SI
.: Angle between p and E is 90°
:. Torque on dipole, = t max = pE sin 90° = q(2b)E x l
.. Torque, t = 2qbE (1)

29. (i)
(1)

In case of constant electric field along Z-direction,


IT
the perpendicular distance between equipotential
surfaces remains same, whereas for field of
increasing magnitude, equipotential surfaces get
closer as we go away from the origin.
In both cases, surfaces be planes parallel to
XY - plane. (1)

32. (i) Refer to Ans. 15. (1)


H
(1)
(ii) .: Every point on X-axis is on equatorial line of
(ii) Equipotential surfaces get closer to each other
electric dipole (system of two unlike charges).
near the point charges as strong electric field is
:. Potential on it is equal to zero.
produced there.
:.No work is done in moving a test charge.
E = - ~v and E cc _ J...-
W = q ~v = q x 0= 0. [.: ~v = 0]
O

~r ~r
Z
For a given equipotential surface, small M
represents strong electric field and vice-versa. (1) B, +q (0, 0, a)
30. Let three point charges y ,
r

" Q (-3, 0, 0)
s.. q2 and q3 have (7,0,0) /0J------y
position vectors r I' r2 and r23 , I
M

/ I
r3, respectively. I I
P -; :
Potential energy of the ~'t-q (0, 0, -a)
charges qi and q2' X
U = _1_. qi q2 X
l2 .. Work done in moving a charge on an
41tEo IrI21 equipotential surface is zero. (1)
=_I_ . ..3J...iL
33. (i) (a) Two equipotential surfaces do not intersect
41tEo Ir2 - rl I each other as normals at intersecting points
Similarly, on two surfaces will give two directions of
electric field which is impossible. (1/2)
U =_I_.3..1..3L =} U =_1_. qi q3
(b) Closely spaced equipotential surfaces
23 3I
41tEo Ir3 - r21 41tEo Ir3 - ril represent strong electric field and
(1) vice- versa. (1/2)
(ii) Refer to Ans. 29 (i). (1)
CHAPTER 2 : Electrostatic Potential And Capacitance 45

34. (i) Let electric potential to be obtained at point P y


Epuipotential
lying on the axis of dipole at distance d from
SUrfr.,s
the centre of the dipole.
>+- 21----+<
--------------------4
--q +q P J--+-+--t--lr--t-+Z

~ d .1
x E
Potential at P due to +q charge
=_I_._q_

R
The equipotential surfaces due to a single point
41tEo (d - ~ charge is represented by concentric spherical
shells of increasing radius, so they are not
Potential at P due to - q charge
equidistant. (1)
1 -q
----- (ill) No, the electric field does not exist tangentially
41tEo (d +~ to an equipotential surface because no work

SI
Total potential at P due to dipole done in moving a charge from one point to other
=_q_[_I_-_I_]
41tEo (d - ~ (d +~
q x 21
41tEo (d2 - /2)
on equipotential surface. This indicates that the
component of electric field along the
equipotential surface is zero. Hence, the
P equipotential surface is perpendicular to field
41tEo (d2 - /2) lines. (1)

where, p = q . 11 36. (i) The figure is shown as below:


if!« d, Equipotential
IT
1 P surfaces
then, V = --2'
41tEo d (1'12)

(ii) Equipotential surfaces of a dipole are as


shown below. Potential of points lies on
perpendicular bisector surface will be zero.

@Y
H
;~:C~tential

~
~- - -- -~

Equipotential surface
.
Equipotentiol surfoces of two identical positive charges

(ii) By definition, electric potential energy of any


charge q placed in the region of electric field is
(1)
O

having potential equal equal to the work done in bringing charge q from
Perpendicular
to zero at all points infinity to that point and given by
bisector of dipole (1'12)
U = qV (112)
35. Any surface that has same electric potential at where, V is the electric potential (as potential at
every point on it is called equipotential surface. infinity is assumed to be zero) where, the charge
(i) Equipotential surface in case of single point
M

q is placed. Now, considering the electric


charge (1) potentials at positions rl and r2 as ~ and V2,
respectively. Therefore, total potential energy of
the system of two charges ql and q2 placed at
points with position vectors rl and r2 in the
region of E is given by U = work done in bringing
E--t-++-t Equipotential charge q from infinity to that position in E is
surfaces equal to work done for charge q 2 from infinity to
that position in E + work done to that of charge q2
at these positions in presence of ql. (1)

(ii) Equipotential surfaces when the electric field i.e. U = ql ~ + q2V2 ~U = _1_ . .3J.iL
is in Z-direction. 41tEo Ir2 - rl I (1/2)
[TOPIC 2] Capacitance
2.1 Conductors and In case of dielectric however, the opposing
field so induced does not exactly cancel the
Insulators external field.
Conductor contains a large number of free charge
- -.:- ~ ++ - - ---- ~ ++-

R
carriers to conduct electricity while insulator does
~ ++ Eo
_ E,n ++ Eo
not contain any free charge carriers to conduct _ l:in
--Eo+E,n=O++ - - --Eo+ Ein .=0++
electrici ty.
Conductor Dielectric
Examples of conductors are metals and graphite. Behovioursof a conductorand a dielectric
Examples of insulators are plastic rod and nylon. in on external electricfield

SI
NOTE Insidea conductor,the electrostaticfieldis zero.
(ii) A net dipole moment is developed by an
external field in either case, whether polar
Free Charges and Bound Charges or non-polar dielectric. The dipole moment
Inside a Conductor per unit volume is called polarisation and
it is denoted by p.
(i) In a metal, the outer (valence) electrons are
free to move. These electrons are free for p=X,E
moving within the metal but not free to leave where, X, is called electric susceptibility
IT
the metal. These free electrons are free of the dielectric medium.
charges inside a conductor and are the cause
of conducting the electricity by conductors. 2.3 Capacitor
(ii) The bound charges are those positive ions
which are made up of nuclei and the bound A capacitoris a device which is used to store
electrostatic potential energy or charge. It
H
electrons remain in their fixed positions.
comprises of two conductors separated by an
NOTE (I) Inside a conductor, the electric field is zero.' insulating medium.
(il) Theinteriorof a conductor can have no excess
charge in static situation. Capacitance of a condurtor
(iil) Electricfieldjust outside a charged conductor
O

is perpendicular to the surface of the If charge q is given to an insulated conductor, it


conductor at every point. leads to increase its electric potential by V such
that q ex V ~ q = CV
2.2 Dielectrics and where, C is known as capacitance of a conductor.
The capacitance depends on the shape, size and
Polarisation
M

geometry of conductor, nature of surrounding


(i) When a conductor is placed in an external medium and presence of other conductor in the
electric field, the free charge carriers adjust neighbourhood of it.
itself in such a way that the electric field . . 1 coulomb
Its SI urnt IS farad (F). Here, 1 farad = ----
due to induced charges and external field I volt
cancel each other and the net field inside
Farad is a very large unit of capacitance. So, I!F is
the conductor is zero.
usually taken.
CHAPTER 2 : Electrostatic Potential And Capacitance 47

Parallel Plate Capacitor Dielectric Strength


The most common among all capacitors is parallel The dielectric strength is equal to that maximum
plate capacitor. It comprises of two metal plates of value of electric field that can exist in a dielectric
area A and separated by distance d filled with air without causing the breakdown of its insulating
or some other dielectric medium. The capacitance property.
of air filled parallel plate capacitor is given by
C - £oA Dielectric and Capacitor

R
0- d
Surface x / Area A
With the advent of dielectric in capacitor, net
electric field inside the dielectric gets reduced,
Charge~
density
1 . I. consequently potential difference across
~~ ++++++++++ (charges disconnected) capacitor decreases.
E

SI
Hence. capacitance of capacitor increases as
j j j j j j j d

Surface

charge
Jc-----;----I y I
C oc ~
V
and new capacitance becomes KC o-

Some Important Points


density -= Earthed
• The capacitance of a parallel plate capacitor
(- 0)
partially filled with a dielectric medium of
When a dielectric of dielectric constant K is filled dielectric constant K is given by
IT
fully between the plates, then
C = 1(.4£0 = KC
o
d

2.4 Dielectric
H
When a dielectric slab is introduced between the
I--t-l
plates of charged capacitor or in the region of I--d--l
electric field, an electric field E p induces inside the
dielectric due to induced charge on dielectric in a C = £oA
(d - t + t/K)
direction opposite to the direction of applied
O

external electric field. Hence, net electric field where, A = area of each plate
inside the dielectric gets reduced to (E 0 - E p)' d = separation between two plates
where, Eo is external electric field. The ratio of t = thickness of dielectric medium
applied external electric field and reduced electric K = dielectric constant of dielectric medium
field is known as dielectric constant K of • Capacitance of spherical capacitor
M

dielectric medium, i.e. K = Eo


Eo - Ep

Dielectric Constant
If C vacuum be capacity of a condenser with vacuum
or air between its plates and Cdielectric be the
capacity with dielectric between the plates, the
dielectric constant K is defined as K Cdielectric. C=41t£Ok(~)
Cvacuum
b-a
Dielectric constant is also known as specific • Capacitance of an isolated spherical conductor
inductive capacity of the dielectric. of radius r is given by
C=41t£or
48 o ehopterwise eBSE Solved Papers PHYSICS

(i) Capacitance of cylindrical capacitor Conductor and Capacitor


L
When a metallic conducting slab is partially filled in
a capacitor, then capacitance of conductor becomes
C = EoA
d- t
-A where, t = thickness of metallic plate and
21tEoKL d = separation between two plates.

R
C
log 0 (bI a) NOTE The dielectric constant of a metallic conductor is infinity.
where,
a = radius of inner coaxial cylinder 2.5 Combination of Capacitors
b = radius of outer coaxial cylinder

SI
Two or more capacitors can be connected by two
L = length of coaxial cylinder different ways:
9iiO Introduction of dielectric medium into the
(charges disconnected) parallel plate
Series Combination of Capacitors
capacitor leads to change in physical The charge on each capacitor is same for any value
quantities as listed below. of capacitance and equal to the charge across the
combination.
Before introduction After introduction
of dielectric slab of dielectric slab
IT
Charge (q) No change (charge - q )

Electric field (E) Decreases ( E' = ~)

Potential difference (V) Decreases ( V' = f) '------flll-----....J


V
H
Capacitance (G) Increases (C' = KG) The potential difference across the combination is
equal to the algebraic sum of potential difference
Electrostatic energy (U) Decreases ( U' = ¥) across each capacitor, i.e. V = VI +V2 +V3
The potential is divided across capacitors in inverse
ratio of their capacitances, i.e.
O

(ill) Introduction of dielectric medium in a


charged capacitor connected with a I I 1
VJ :V2 :V3 =-:-:-
battery. CI C2 C3
The equivalent capacitance is given by
Before After introduction of
introduction of 1 1 1 I
dielectric slab -=-+-+-
dielectric slab
M

C CI C2 C3

Potential difference No change, Potential The equivalent capacitance of n identical capacitors


(V) difference (V) connected in series each of capacitance C is given by
C
Capacitance (C) Increases, Capacitance C, =-
(C'=KG) n

Charge (q) Increases, Charge q' = qK Parallel Combination of Capacitors


Electric field (E) No change, Electric field The potential difference across each capacitor is
E'=E same for any value of capacitance and equal to the
Electrostatic Increases, Electrostatic potential difference across the combination.
potential energy (U) energy U' = KU
q, +_ C,
+-
v
PREVIOUS YEARS'
EXAMINATION QUESTIONS
C TOPIC 2
+-
o 1 Mark Questions

R
v

v 1. Predict the polarity of the capacitor in


The total charge on combination is equal to the the situation described below:
algebraic sum of charges on each capacitor, i.e.

SI
q = q, + q2 + q3
The equivalent capacitance (C) is given by I_s _N-.Jlm - m -:------j_S __ N 1
C =C, +C2 +C3 c
The total charge on the capacitors is divided in the
ratio of their capacitances,
All India 2017
i.e. q ex C ~ q,: q2 :q3 = C, : C 2 : C 3
The equivalent capacitance of n identical capacitors 2. Why does current in steady state not
flow in a capacitor connected across a
IT
connected in parallel combination is Cp = nC.
battery? However, momentary current
2.6 Energy Stored in a Capacitor does flow during charging or discharging
of the capacitor. Explain. All India 2017
Electrostatic energy stored in a (parallel plate)
3. The given graph shows the variation of
capacitor is given by
charge q versus potential difference V for
l
H
1 2 I two capacitors ~and C2. Both the
U=-CV =-=-qV
2 2C 2 capacitors have same plate separation
where, q = charge on capacitor, but plate area of C2 is greater than that
~. Which line (A or B) corresponds to C1
C = capacitance,
and why? All India 2014C
O

V = potential difference across capacitor.


The energy stored per unit volume in an electric A
field E is known as energy density. It is given by
I 2 q
UE = -£oE B
2
M

Common Potential v
When two capacitors of different potentials are
4. Distinguish between a dielectric and a
connected by a conducting wire, then charge flows
conductor. Delhi 2012
from capacitor at high potential to the capacitor at
low potential. This flow of charge continues till 5. Define the dielectric constant of a
their potentials become equal, this equal potential medium. What is its unit? Deihl 2011C
is called common potential. 6. A metal plate is introduced between the
Common potential, V
.
= C 'Iv: +C V
2 2
plates of a charged parallel plate
. C, +C2 capacitor. What is its effect on the
capacitance of the capacitor? Farelgn 2009
50 o Chapterwise CBSE Solved Papers PHYSICS

o 2 Marks Questions fill the gap between the plates. Find the
ratio of
7. Calculate the potential difference and the (i) the net capacitance and
energy stored in the capacitor C2 in the (ii) the energies stored in the combination
circuit shown in the figure. Given before and after the introduction of the
potential at A is 90 V, C; = 20 Il F, dielectric slab. Delhi 2014C
C2 = 30 Il F and c; = 151lF. k
--I r------1 ~ ~
1

R
A

C1 C2 C3
B

-+ ~.) 1:
All Indio 2015
V T I C
1 T C
2

SI
8. A parallel plate capacitor of capacitance C
is charged to a potential V. It is then 11. Find the charge on the capacitor as
connected to another uncharged capacitor shown in the circuit. Foreign 2014
having the same capacitance. Find out the 6IJ.F
II
ratio of the energy stored in the combined II

system to that stored initially in the single


capacitor. All Indio 2014
9. Two parallel plate capacitors of 10n 20n
IT
capacitances C; and C2 such that C1 = 2C2
are connected across a battery of V volt as
shown in the figure. Initially, the key (k) is I
I
kept closed to fully charge the capacitors. 2V
The key is now thrown open and a
12. A slab of material of dielectric constant K
dielectric slab of dielectric constant K is
has the same area as that of the plates of
H
inserted in the two capacitors to
a parallel plate capacitor, but has the
completely fill the gap between the plates.
thickness d/2, where d is the separation
Find the ratio of
between the plates. Find out the
(i) the net capacitance and expression for its capacitance when the
(ii) the energies stored in the combination
O

slab is inserted between the plates of the


before and after the introduction of the capacitor. Delhi 2013
dielectric slab. Delhi 2014C 13. Determine the
k
potential difference
~.) 1: 1 across the plates of
M

the capacitor C1 of

V T I C
1 T C
2
the network shown
in the figure.
(assume, E2 > ~)
10. Two parallel plate capacitors of All Indio 2013
capacitances C1and C2 such that C1 = C2/ 2 14. A parallel plate capacitor, each of plate
are connected across a battery of V volts as area A and separation d between the two
shown in the figure. Initially, the key (k) is plates, is charged with charges +Q and
kept closed to fully charge the capacitors. -Q on the two plates. Deduce the
The key is now thrown open and a expression for the energy stored in
dielectric slab of dielectric constant K is capacitor. Foreign 2013
inserted in the two capacitors to completely
CHAPTER 2 : Electrostatic Potential And Capacitance 51

15. A network of four completely the space between the plates of


capacitors, each of the two capacitors. How will the (i) charge
capacitance 15~F, is and (ii) potential difference between the
connected across a plates of the capacitors be affected after
battery of 100 V, as the slabs are inserted?
shown in the figure. Find
the (i) net capacitance
1 I -{~
and (ii) the charge on the

r C'I_ r·

R
capacitor C4• All India 2012C 2 IlF C F
16. Deduce the expression for the electrostatic
V _ _'
energy stored in a capacitor of capacitance
C and having charge Q.
All India 2011; Delhi 2011

SI
How will the
20. What is the area of the plates of 2 F
(i) energy stored and parallel plate capacitor having separation
(ii) the electric field inside the capacitor between the plates is 0.5 cm?Alllndia 2011
be affected when it is completely
21. Two identical
filled with a dielectric material of
dielectric constant K? All India 2012 parallel plate (air)
capacitors CI and
T T
1/2
17. 1 ~F capacitance connected to a battery of C2 have
6 V. Initially switch S is closed. After capacitance C
1 1
IT
sometime S is left open and dielectric each. The space
slabs of dielectric constant K = 3 are between their
inserted to fill completely the space plates is now filled I-- d ----l
between the plates of the two capacitors. with dielectrics as
How will the (i) charge and (ii) potential shown in the figure. If the two capacitors
difference between the plates of the still have equal capacitance, then obtain
H
the relation between dielectric constants
capacitors be affected after the slabs are
K, KI and K2. Foreign 2011
inserted? Delhi 2011
22. You are given an
air filled parallel
T
~~
T T plate capacitor CI.
O

6V C1 1 IlF 1 IlF
The space
between its plates
18. Net capacitance of three identical is now filled with
capacitors in series is 11lF. What will be slabs of dielectric
their net capacitance, if connected in constants KI and
M

K 2 as shown in I-- d ----l


parallel?
Find the ratio of energy stored in these figure. Find the capacitance of the
two configurations, if they are both capacitor C2 if area of the plates is A and
connected to the same source. All India 2011 distance between the plates is d. Foreign 2011

19. Figure shows two identical capacitors C1 23. Figure shows a sheet of
aluminium foil of ~
and C2, each of 2 ~F capacitance,
negligible thickness placed
connected to a battery of 5 V. Initially,
between the plates of a
switch S is closed. After sometime, S is
capacitor. How will its
left open and dielectric slabs of dielectric
constant K = 5 are inserted to fill capacitance be affected, if
(i) the foil is electrically insulated?
52 o Chapterwise cast Solved Papers PHYSICS

(ii) the foil is connected to the upper


plate with a conducting wire?
Find the value of the
following -IIJ+
Foreign 2011 (i) energy stored in
24. Distinguish between polar and non-polar 1211F capacitor
dielectric. All Indio 2010 C (ii) energy stored in 6~"F
311F capacitor
o 3 M'crks Questions 12JlF
(iii) total energy drawn from the battery

R
Foreign 2016
25. Two identical
parallel plate 29. Find the ratio of the potential differences
capacitors A that must be applied across the parallel
andB are and series combination of two capacitors C1
connected to a and C2 with their capacitances in the ratio

SI
battery of V 1 : 2, so that the energy stored in these two
volts with the switch S is closed. The switch cases becomes the same. All Indio 2016
is now opened and the free space between 30. Two capacitors of unknown capacitances C1
the plates of the capacitors is filled with a and C2 are connected first in series and
dielectric of dielectric constant K. Find the then in parallel across a battery of 100 V. If
ratio of the total electrostatic energy stored the energy stored in the two combinations
in both capacitors before and after the is 0.045 J and 0.25 J respectively, then
introduction of the dielectric. All Indio 2017 determine the value of C1 and C2. Also,
IT
26. A 12 pF capacitor is connected to a 50 V calculate the charge on each capacitor in
battery. How much electrostatic energy is parallel combination. All Indio 2015
stored in the capacitor? If another 31. Calculate the potential difference and the
capacitor of 6 pF is connected in series energy stored in the capacitor C2 in the
with it with the same battery connected circuit shown in the figure. Given
H
across the combination, find the charge potential at A is 90 V, c;. = 2011F,
stored and potential difference across C2 = 30 I1Fand Ca = 1511F. Delhi 2015
each capacitor. Oelhi 2017
27. Two parallel plate
capacitors X and Y CJ X y
A--1~~~
C1 C2 C3 -=-
O

have the same area of 32. (i) Obtain the expression for the energy
plates and same st6red per unit volume in a charged
separation between + - parallel plate capacitor.
them, X has air 15 V (ii) The electric field inside a parallel plate
between the plates capacitor is E. Find the amount of work
while Y contains a dielectric medium of
M

done in moving a charge q over a closed


e, = 4. rectangular loop abeda. Delhi 2014
(i) Calculate the capacitance of each
capacitor, if equivalent capacitance 33. (i) Derive the expression for the
of the combination is 411F. capacitance of a parallel plate
(ii) Calculate the potential difference capacitor having plate area A and
between the plates of X and Y. plate separation d.
(iii) Estimate the ratio of electrostatic (ii) Two charged spherical conductors of
energy stored in X and Y. Oelhi 2016 radii ~ and R2 when connected by a
28. In the following arrangement of conducting plate respectively. Find
capacitors, the energy stored in the 611F the ratio of their surface charge
densities in terms of their radii.
capacitor is E.
Delhi 2014
CHAPTER 2 : Electrostatic Potential And Capacitance 53

34. In a parallel plate capacitor with air 38. A network of four capacitors each of 121lF
between the plates, each plate has an capacitance, if connected to a 500V supply
area of 6 x 10-3 m2 and the separation as shown in the figure.
between the plate is 3 mm. C2
(i) Calculate the capacitance of the
capacitor.
(ii) If this capacitor is connected to 100 V

R
supply, what would be the change on
each plate?
(iii) How would charge on the plates be Determine
affected if a 3 mm thick mica sheet of (i) the equivalent capacitance of the
K = 6 is inserted between the plates network and

SI
while the voltage supply remains (ii) the charge on each capacitor.
connected? Foreign 2014 HOTS; All India 2010

35. A sphere 81 of radius r1 encloses a net 39. A parallel plate capacitor is charged by a
charge Q. If there is another concentric battery. After sometime, the battery is
sphere 82 of radius r2(r2 > r1) enclosing disconnected and a dielectric slab with its
charge 2Q. thickness equal to the plate separation is
inserted between the plates. How will
(i) Find the ratio of
(i) the capacitances of the capacitor,
the electric flux
IT
through sphere 81 S2
(ii) potential difference between the
plates and
and8z·
(iii) the energy stored in the capacitors be
(ii) How will the affected? Justify your answer in each
electric flux case. Delhi 2010
through sphere 81 40. A parallel plate capacitor, each with plate
change, if a medium of dielectric area A and separation d is charged to a
H
constant 5 is introduced in the space potential difference V. The battery used to
inside 81 in place of air? All India 2014 charge it remains connected. A dielectric
36. A capacitor of unknown capacitance is slab of thickness d and dielectric constant
connected across a battery of V volt. The K is now placed between the plates.
What change if any will take place in
O

charge stored in it is 360llC . When


potential across the capacitor is reduced (i) charge on plates?
by 120 V, the charge stored in it becomes (ii) electric field intensity between the
120IlC. plates?
Calculate (iii) capacitance of the capacitor?
(i) the potential Vand the unknown
M

Justify your answer in each case. Delhi 2010


capacitance C.
(ii) what will be the charge stored in the 41. A parallel plate capacitor is charged to a
capacitor, if the voltage applied had potential difference V by a DC source. The
increased by 120 V? Delhi 2013 capacitor is then disconnected from the
37. A capacitor of 200 pF is charged by a source. If the distance between the plates
300 V battery. The battery is then is doubled, state with reason, how the
disconnected and the charged capacitor is following will change? Delhi 2010
connected to another uncharged capacitor (i) Electric field between the plates
of 100 pF. Calculate the difference (ii) Capacitance
between the final energy stored in the (iii) Energy stored in the capacitor.
combined system and the initial energy
stored in the single capacitor. Foreign 2012
54 o ehopterwise eBSE Solved Papers PHYSICS

42. Show that the capacitance of a spherical Determine the


conductor is 41teo times the radius of the (i) equivalent capacitance of the network.
spherical conductor. Delhi 2010 (ii) charge on each capacitor. Delhi 2010C
43. Find the ratio of the potential differences 47. A parallel plate capacitor is charged by a
that must be applied across the parallel battery. After sometime, the battery is
and the series combination of two identical disconnected and a dielectric slab of
capacitors, so that the energy stored in the dielectric constant K is inserted between

R
two cases becomes the same. Foreign 2010 the plates. How would
44. (i) How is the electric field due to a (i) the electric field between the plates?
charged parallel plate capacitor (ii) the energy stored in the capacitor be
affected when a dielectric slab is affected? Justify your answer.
inserted between the plates fully Afllndia 2009

SI
occupying the intervening region? 48. Three identical capacitors C1 , C2 and c,
of
(ii) A slab of material of dielectric capacitances 6 ~F each are connected to a
constant K has the same area as the

:r---I
12 V battery as shown below:
plates of a parallel plate capacitor

C
but has thickness.!. d, where d is the
2
separation between the plates. Find c~C'
the expression for the capacitance
IT
when the slab is inserted between
the plates. Foreign 2010 Find
45. (i) Plot a graph comparing the variation (i) the charge on each capacitor
of potential V and electric field E due (ii) the equivalent capacitances of the
to a point charge Q as a function of network
distance R from the point charge. (iii) the energy stored in the network of
H
(ii) Find the ratio of the potential capacitors. Delhi 2009C
differences that must be applied 49. The equivalent capacitance of the
across the parallel and the series combination between points A and Bin
combination of two capacitors, C1 and the given figure is 4 ~F.
O

C2 with their capacitances in the A - II II -8


ratio 1 : 2, so that the energy stored 20~F C
in the two cases becomes the same. (i) Calculate the capacitance of the
Foreign 2010 capacitor C.
(ii) Calculate the charge on each
M

46. Four capacitors of values 6 ~F, 6 ~F, 6 ~F


capacitor if a 12 V battery is
and 2 ~F are connected to a 6 V battery as
connected across terminals A and B.
shown in the figure.
(iii) What will be the potential drop
6 ~F
...---410::---; It----; across each capacitor? Delhi 2009
50. Two parallel plate X y
capacitors X and Y have [.]
the same area of the
plates and same
separation between them. +_
X has air between the 12 V
plates while Y contains a
dielectric medium of e, = 4.
ra'
CHAPTER 2 : Electrostatic Potential And Capacitance 55

(i) Calculate the capacitance of each (ii)


capacitor if equivalent of the
A thin metallic
spherical shell of 2'6
combination is 4JlF. radius R carries a Q' C
charge Q on its -
(ii) Calculate the potential difference surface. A point 2
between the plates of X and Y.
charge Q is placed at its centre C and
(iii) What is the ratio of electrostatic 2
energy stored in X and Y? Deihl 2009 an other charge + 2 Q is placed
51. A system of outside the shell at a distance x from
the centre as shown in the figure.
capacitors

R
Find
connected as
(a) the force on the charge at the centre
shown in the of shell and at the point A,
figure has a
(b) the electric flux through the shell.
total energy of
All India 2015

SI
160 mJ stored
in it. Obtain the + 54. (i) Derive the expression for the energy
value of the ---- ..•
200 V stored in parallel plate capacitor .
equivalent Hence, obtain the expression for the
capacitance of this system and the value energy density of the electric field.
of Z. All India 2009C (ii) A fully charged parallel plate capacitor
is connected across an uncharged
[ZI 5 Marks Questions identical capacitor. Show that the
IT
energy stored in the combination is
52. (i) If two similar large plates, each of area less than stored initially in the single
A having surface charge densities + o capacitor. Delhi 2014
and - o are separated by a distance d
in air, find the expression for 55. (i) Obtain the expression for the
(a) field at points between the two potential due to an electric dipole of
plates and on outer side of the dipole moment P at a point x on the
H
plates. Specify the direction of the axial line.
field in each case. (ii) Two identical capacitors of plate
(b) the potential difference between the dimensions l x b and plate separation
plates. d have dielectric slabs filled in
between the space of the plates as
O

(c) the capacitance of the capacitor so


shown in the figure.

~-.;
formed. ~I~ 1 ~I
(ii) Two metallic spheres of radii Rand
2R are charged, so that both of these
f :
have same surface charge density cr.
If they are connected to each other r~~>~~
M

with a conducting wire, in which


direction will the charge flow and why?
All India 2016
~~
53. (i) Explain using suitable diagrams, the
difference in the behaviour of a
(a) conductor and
(b) dielectric in the presence of
i".~:,)
I+-- 1/2 ~I• 1/2 --+l
external electric field. Define the Obtain the relation between
terms polarisation of a dielectric dielectric constants K, s, and K2•
and write its relation with All India 2013
susceptibility.
56 o Chnpterwlse eBSE Solved Papers PHYSICS

56. (i) A parallel plate capacitor is charged by From Lenz's law, induced current produces same
a battery to a potential. The battery is polarity as that of approaching pole. So, plate A
disconnected and a dielectric slab is will have +ve polarity and plate B will have -ve
inserted to completely fill the space polarity. (1)

between the plates. 2. In steady state, electric flux between plates of a


How will capacitor is constant.
So, displacement current is,
[II
(a) its capacitance
id =Eo dellE and dcj>E =0 =? id =0
( . (b) electric field between the plates and
dt dt

R
(c) energy stored in the capacitor be
So, there is no current between plates when steady
affected? Justify your answer
state is reached.
giving necessary mathematical
expressions for each case. During charging, flux is increasing.

(ii) (a) Draw the electric field lines due to a •• dcj>E 0 *


dt

SI
conducting sphere.
Hence, a displacement current exists in the
(b) Draw the electric field lines due to a
.
capaotor w hi ch iIS ld = Eo -.dcj>E
dipole. All India 2011 dt (1)
57. (i) Deduce the expression for the energy
3. Line B corresponds 'to C1 because slope (q versus V)
stored in a charged capacitor.
of B is less than slope of A. (1)
(ii) Show that the effective capacitances
C of a series combination of three 4. Dielectrics are non-conductors and do not have
capacitors ~ , C2 and Ca is given by free electrons at all. While conductor has free
electrons which makes it able to pass the
IT
C= ~ C2 Ca .
electricity through it. (1)
~ C2 + C2 c,
+ Ca ~ All India 2010C
5. Dielectric When a dielectric slab is introduced
58. (i) Show that in a parallel plate between the plates of charged capacitor or in the
capacitor, if the medium between the region of electric field, an electric field E p induces
plates of a capacitor is filled with an inside the dielectric due to induced charge on
insulating substance of dielectric dielectric in a direction opposite to the direction
H
constant K, its capacitance increases. of applied external electric field. Hence, net
(ii) Deduce the expression for the energy electric field inside the dielectric gets reduced to
stored in a capacitor of capacitance C Eo - Ep, where Eo is external electric field. The
ratio of applied external electric field and reduced
with charge Q. Delhi 20D9C ' "
electric field is known as dielectric constant K
O

59. A small sphere of radius a carrying a . of dielectric medium, i.e. K = Eo


positive charge q is placed concentrically Eo - Ep
inside a large hollow conducting shell of and it is a dimensionless quantity. (1)
radius b (b > a). This outer shell has charge
Q on it. Show that if these spheres are 6. If a metal plate is introduced between the plates of
a charged parallel plate capacitor, then capacitance
M

connected by a conducting wire, charge will


r, always flow from the inner sphere to the of parallel plate capacitor will become infinite. (1)

, outer sphere irrespective of the magnitude 7. For a series combination of three capacitors Cl' C2
of the two charges. All India 2009 and C3' the equivalent capacitance Ceq will be

o Explanations 1
-=-+-+-
Ceq
11111
C1 C2 C3
=? -=-+-+-
Ceq 20
II
30 15
1. The polarity of the capacitor shown below. C1 =20 JlF C2=30 JlF

~-----o-~----~
-+ -B

,
__
C~ 90V
CHAPTER 2 : Electrostatic Potential And Capacitance 57

~_1_ = 3+ 2+ 4 ~C = 601lF= 20JlF Energy stored in the combination after

~:J
Ceq 60 eq 9 3 introduction of dielectric slab.
(1)

Charge on equivalent capacitor C d ... (v)


20 ~ Ufin•1 = 3KC
x90 2
Q=C eq V=-x10-6
3 :j: ::: Ratio of energies stored
~ Q= 600llC
Uiniti.1.: [from Eqs. (iv) and (v)]
Charge on each capacitor is
same as they are in series. 90 V Ulin•1 (1)

R
Now, potential drop across C2 10. Given, C1 = C2 •.. (i)
2
V = ~ = 600 X 10-6 = 20V
2
C2 30 X 10-6 Hence, Cimtial= C1 + C2 = C2 + C2 = 3C2 •.. (ii)
2 2
Hence, work done stored as electric potential Net capacitance after filling the gap with

SI
energy U of capacitor is U = ~c2vf diIe 1ectnc. KC
, initial-- --
3KC2
2
2 (2)
U = ~ x 30 X 10-6 x (20)2 = 6 X 10-3 J 11. Total current through the circuit is given by
2 ru 1= VIR
8. Let q be the charge on the charged capacitor. Here, V=2V ~ R=(10+20)n=30n
2
:. Energy stored in it is given by U = !L .. 1=2 =~A
2C 30 15
When another uncharged similar capacitor is Voltage across IOn resistor =1(10) = 10/15 = '!:V
IT
connected, then the net capacitance of the system 3
is given by C' = 2C (1) Charge on the capacitor is given by
The charge on the system remains constant. So, Q = CV =(6 x10-, x 2/3 = 41lC (2)
the energy stored in the system is given by 12. Initially, when there is a vacuum between two
, q2 q2
U =-=- [':C'=2C] plates, then capacitance of the plate is Co= Eo A,
2C' 4C d
H
Thus, the required ratio is given by where, A is the area of parallel plates.
U' _ q2 14C _ 1 Suppose that the capacitor is connected to a
U- q2 12C - '2 (1)
battery.jm electric field Eo is produced. Now, if
we insert the dielectric slab of thickness t=dI2,
9. (i) Given, C1 = 2C2 ... (i) the electric field reduces to E .
O

Net capacitance before filling the gap with Now, the gap between plates is divided in two
dielectric slab is given by pans, for distance t, there is electric field E and
[from Eq. (i)] fur the remaining distance (d - t) the electric field
Ciniti.1=C1 + C2
... (ii) is Eo. (1)
Ciniti.1=2C2 + C2 = 3C2
If V be the potential difference between the plates
Net capacitance after filling the gap with
M

of the capacitor, then V =Et + Eo (d - t)


dielectric slab of electric constant K
Ciniti.1=KC1 + KC2 =K(C1 + C2) [from Eq. (ii)] V=E: + E~d=~(E+Eo) [-: t=~J

Clin•1=3KC2 ••• (iii)


~ V=~(;+Eo)
Ratio of net capacitance is given by

Ciniti.1= 3C2 = ~ [from Eqs. (ii) and (iii)]


Clin•1 3KC2 K (1)
=~(K+1) [AS, ~=Kl
(ii) Energy stored in the combination before Now, Eo=!!....=-q- ~V=~.-q-(K+1)
introduction of dielectric slab Eo EoA 2K EoA

d ... (iv)
Ne know that, C=!L= 2KEoA
Uiniti.1= 3C V d(K +1) (1)
2
58 o ehapterwise eSSE Solved Papers PHYSICS

13. Potential difference, Here C1, C2 and C3 are in series, hence their
equivalent capacitance C' is given by
-q +EI-3....-E =0 or 3....+3....=E
-E2
2 1 1 1 1 1

r~
C1 C2 C1 C2 (1) -=-+-+-
C' C1 C2 C3
Redrawing the circuit as shown below
1 1
-=-+-+-
1 1
C'
r;g
C' 15 15 15

El~~_r

R
C' = ~IlF
3
~ C'=5IlF
q Since, C' and C4 are in
Vi = -q and V2= +q parallel,
Now,
C1 C2 :.C = C' + C4 = 51l + 151l = 20ll (1)

SI
(1)
(ii) Since, C' and C4 are in parallel potential

-j~l-
14. The work done in +0 difference across both of them is 100 V.
charging the
capacitor is stored
as its electrical
potential energy.
=
:. Charge across C4 is ~ = C4 X V = C4 X 100C
15 X 10-6 x 100
= 15xlO-4C
Hence considering ~ =1.5mC (1)
a capacitor of
capacitance C, initially whose two plates are 16. Expression for the energy stored in a capacitor:
IT
uncharged, let Q and -Q are charges on the two If we consider a capacitor of capacitance C,
plates and produces a uniform electric field, initially whose two plates are uncharged the
positive charge will be transferred from plate 2 to
E = ~ between the plates and a potential
plate 1 bit by bit. During this process, the
Eo (1)
pote~tiabPifference between the two plates will
difference v=i ... (i)
beY =-.
C
If a charge dq is transported in steps from negative If a smafi' additional charge det be transferred
H
charged plate to positive charged plate, till charges from plate 2 to plate 1, the work done will be,
rises to + Q and -Q, then
dW = V'·det = Q' . dQ'
work done, dW = dq. V ... (ii)
,) C
From Eqs. (i) and (ii), we get The total work done in transferring a charge Q
f dW
O

from plate 2 to plate 1 will be


dW = dq( ~) ~ W = ',T ,"

This work done is stored as electrical potential U of W = f dW = foQQ'-'C dQ


,

the capacitor,

U = W =
Q
q
I c·
dq
Q2
= 2C
=[~;I=~'~
M

Q2 1 2 1 .
U = - = -' CV = - QV [.: Q = CV] This work done is stored as electric potential
2C 2 2 (1)
energy U of the capacitor,
15. (i) According to the diagram given in the 1 Q2 1 2
question U=-·-=-·CV
2 C 2
(i) Energy stored will be decreased or energy
1 II 1 stored will become ~tirnes the initial energy.
C1~t-JC3 K
(ii) Electric field would decrease or

L1~V-l E'=~
K
CHAPTER 2 : Electrostatic Potential And Capacitance 59

17. According to the diagram are when the switch S is In parallel combination,
closed, the two capacitors CI and C2 in parallel will Cp = nC = 3 x 3 = 9 ~F
be charged by the same potential difference V.
c, = 9~F (1)
S
For same voltage, U cc C
~ Us = Cs ~
-=- 6V
Up c,
T_ Us _ 1 _ 1
Up - (3)2 -"9
~

R
So, charge on capacitor CI or Us : Up = 1 : 9 (1)
ql = CIV= 1 x 6 = 6~ ... (i)
19. When a dielectric medium of dielectric constant
and charge on capacitor C2 K is introduced,
q2 = C2V=1 x 6= 6~ ... (ii)

SI
(i) in an isolated (not connected with battery)
Hence total q = ql + q2 = 6 + 6 = 12 ~ (1) capacitor, then total charge on capacitor
When switch S is opened and dielectric is remains same.
introduced. Then, (ii) in a capacitor connected with battery, then
s potential difference across the capacitor
remains same as that of potential difference
across battery.
rzz: 6V e', A B Two identical capacitors CI and C2 get fully
charged with 5 V battery initially.
IT
So, the charge and potential difference on both
Capacity of both the capacitors becomes K times capacitors becomes
i. e. C{ = C~ = KC = 3 x 1 = 3~ (as CI '= C2 ') q = CV = 2 X 10-6 X 5 V = 10 ~C
Capacitor A remains connected to battery
and V = 5V
.. V'I = 'V = 6V
On introduction of dielectric medium of K = 5. (1/2)
q{ = KtJI = 3 x 6~ = 18~
H
For C1 (continue to be connected with battery)
Capacitor B becomes isolated
:. q~ = q2 or C~V; = C2V2 or (KG)V; = CV potential difference of CI ' (Vi = 5 V
Capacitance of C; = KC = 5 x 2~F = 10 ~
or V2=(f)=~=2V Charge, q' = C'V' = (10~F) (5 V) = 50 ~C
(1)
For C2 (disconnected with battery)
18. If n identical capacitors, each of capacitance Care
O

Charge, q' = q = 10 ~C
connected in series combination give equivalent
Potential difference,
capacitance, C, = ~ and when connected in
n V'=~=~=l V
K 5 (1Y2)
parallel combination, then equivalent
capacitance, Cp': nC 20. According to the question, separation between
M

Also, for same voltage, energy stored in the the plates = d = 0.5 em = 0.5 x 1O-2m and
capacitor is given by capacitance C = 2 F
U = ~CV2 [for constant] Dielectric constant
2 d = O.5cm = 0.5 x 10-2 m .er
UocC Eo = 8.854 X 10-12 2 -lm-2

c, =1 ~F [·:n = 3] C= EoA
d
. com binati
In senes matlon, Cs =-C 2
n A = Cd = 2 x 0.5 X 10-
Eo 8.854 x 10 12
In parallel combination, Cp = nC
= 1.13 X 109 m2 (2)
According to the problem,
C = n C, = 3 x 1 ~F = 3~F 21. The capacity of condenser is proportional to the
For each capacitor, area and inversely proportional to the distance
60 IZl Chopterwise CBSE Solved Papers PHYSICS

between its plates. If a medium of dielectric which two plates of each capacitor have
constant K is filled in the space between the . separation half of the original separation.
plates, its capacity becomes K times the capacity Thus, new capacitance of each capacitor
when there is air between the plates.
After inserting the dielectric medium, let their
C'=2C [-: coc~]
capacitances become C; and C;. C and C' are in series
C; = KC ... (i) (112) C =2CX2C=C
net 2C + 2C
For Cz C' - x, Eo (A/2)
+
_K.=...2E-,,-o.:....(A...:../2....:..)

R
2-
d d Cnet = C (Original capacitor) (1)
C2 acts as if two capacitors each of area A/2and (ii) System reduces to a capacitor whose
separation d are connected in parallel separation reduces to half of original one.
combination.
:. New capacitance, C' = 2C (1)
C; = EoA (Kl + K2)

SI
d 2 2
24. Polar dielectrics
A polar molecule which has permanent electric
C; = C (Kl : K2) ... (ii) dipole moment (p) in absence of electric field

[-: C - Eo
d
A]
(1/2)
also polar molecules are randomly oriented.
e.g. Water, alcohol. Hel. NH}
Non-polar dielectrics
According to the problem,
A non-polar molecule having zero dipole moment
C; = C; in its normal state.
IT
Non-polar molecules have symmetrical shapes
e.g. any non-conducting material.
(1)
25. The given figure is shown below.
22. After introduction of dielectric medium of
dielectric constants K, and K2, capacitor acts as if
it consists of two capacitors, each having plates of
H
area A and separation ~ connected in series
2
combination for
EoA
C1 =-- ... (i) When switch S is closed, the potential difference
d
I I I across capacitors A and B are same
O

i.e. V = QA = QB
C;=(KIEOA)+ (K2EOA) C C
d/2 d/2 (1) Initial charges on capacitors

-=---I I (I--+- I) QA = QB = CV •
When the dielectric is introduced, the new
(1)
M

C2 (E~A) 2Kl 2K2


capacitance of either capacitor
I _
C'=KC
As switch S is opened, the potential difference
across capacitor A remains same (V volts).
Let potential difference across capacitor B be V'.
When dielectric is introduced with switch S open
(i.e. battery disconnected), the charges on
capacitor B remains unchanged, so
The capacitors will be in series. (1)
QB = CV= C'V'
V'=!:.. V =!volt
23. (i) The system will be equivalent to two identical c' K (1)
capacitors connected in series combination in
CHAPTER 2 : Electrostatic Potential And Capocitonce 61

Initial energy of both capacitors (ii) Charge flowing through the capacitor is given by
U = ~CV2 + ~CV2 =
122
cv2 q = CV = -4C x 15 = --4x 5
x 15 = 60 ~
5 5
Final energy of both capacitors Now, let the potential difference between
UI = ~C'V2 + ~C'V'2
2 2
= ~(KC) V2
2
+ ~(KC)
2
(~)2 K
plates of capacitors X and Yare Vx and,Vy,
respectively. .

1CV 2 [ 11
So, Vx = !L = 60 = 12 V and Vy = !L = 60 = 3 V
="2 K+"KJ c, 5 c, 2,~ (1)

R
(iii) Electrostatic energy stored in capacitance
2
= ~ cv ( K~+ 1)
X(Ex) = ~Cv; I ••• (i)
2
U, _ CV 2K
Similarly for Y,Ey =~4CV; ... (ii)
2 2
UI ~CV2 (K~+I) - K +1

SI
(1)
From Eqs. (i) and (ii), we get
1 2
. E x V 2
26. Energy stored in capacitor =~ C1V2 RatIO = -L = _2-CV
__ = _x_ = 12x12
= 4: 1
2
Ey ~4CV2 4V; 4x3x3
= ~ x 12 X 10-12 x (50)2 J 2 y (1)
2
28. (i) As given in the question, energy of the 6 ~
= 6x 25 X 10-10 J = 15 x 1O-9J capacitor is E. Let V be the potential difference
With other capacitor 6 pF in series. along the capacitor of capacitance 6 ~F. From
IT
. C xC 6 x 12 the mathematical formula,
Total capacitance (C) = _I __ 2 = -- pF
C1 + C2 6 + 12 Since, ~ CV2 = E
_ 12 x 6 _ 4 F 2
---- P
18 n) ~X6xlO-6xV2=E=>V2=~xI06 ... (i)
2 3
Charge stored in each capacitor is same and is
given by Since, potential is same for parallel
H
connection, the potential through 12 ~
Q = CV = 4 X 10-12 X 50 C = 2 X 1O-IOC capacitor is also V. Hence, energy of lzu
Each of the capacitors will have charge equal to Q capacitor is
= 2 X 1O-IOC (1) . EJ2 = ~ x 12x 10-6 x V2 [From Eq. (i)]
Potential on capacitors with capacitance 12 pF is
2
O

10
=!l = 2 X 10- V = 16.67V = ~ x 12 X 10-6 x ~ X 106 = 2E
C1 12 x 10 12 Y 3 m
(ii) Since, charge remains constant in series, the
Potential on capacitor with capacitance 6 pF is
10 charge on 6~ and 12 ~ capacitors combined
2 x 10- V = 33.33 V will be equal to the charge on 3 ~ capacitor.
6 x 10-12
M

n) Using the formula, Q = CV, we can write


27. According to question, let the capacitance of X be => (6 + 12) x 10-6 x V = 3 X 10-6 X V'
C, so capacitance of Y = e, C = 4C [.: e, = 4] V'=6V
(I)
1
E qmva
. Ient capacitance
. Cx4C
= -- Using Eq. (i) and squaring both sides, we get
C+4C
V'2= 12V2 => V'2 = 12E X 106
(X and Yare in series)
2
= -4C => -4C an d It i gIven t h at -4C = 4 ~
it IS :. E3 = ~ x 3 X 10-6 x 12E X 106 = 18E
5C 5 5 2 m
So, 4C = 20~ = capacitance of Y (iii) Total energy drawn from battery is
. 20
Capacitance of X = C = - = 5 ~ Etotal = E + EJ2 + E3
4 (1) = E + 2E + 18E = 21E
62 o Chopterwise CBSE Solved Papers PHYSICS

29. Total energy stored in series or parallel ~ CI - C2 = 2.64 x 10- 5 ... {ii)
combination of capacitors is equal to the sum of
On solving Eqs. (i) and (ii). we get
energies stored in individual capacitors. In parallel
combination energy stored in the capacitor CI = 35 JlF and C2 = 15 JlF ('I. + 'I.)
1 2 1
= -CIl'J. + -C2VJ
2
..• {i) QI =CIV=35XlO-6x100=35xIO-4C (112)
2 2 6 4
Q2=C2V=15XlO- x100=15xIO- C (112)
In series combination energy stored in the
capacitor 31. Consider the given figure
... (ii) 20 JlF 30JlF 15 JlF

R
(1)
A---ll---ll---l ~
According to the question, energy in both the cases C1 C2 C3 -=-
is same so, Given, CI = 20JlF, C2 = 30JlF, C3 = 15JlF
~C+~C)V,2= CIC2 V2 Potential at A = 90 V
(2 I 2(CI + C2) 2

SI
2 2 I As, we can see that capacitor C3 is earthed,
therefore, potential across C3 will be zero.
VI2 _ CI C2 X 2 l'J. _ ~CI C2
vi - --~-"----~
2(CI + C2){CI + C2) V2 - CI + C2
Since, capacitor CI' C2 and C3 are connected
series, therefore
in

S. = ~ ~ C2 = 2CI _1_= J.... + J....+ J....~ _1_=J....+ J....+ ~


But,
C2 2 Ceq CI C2 C3 Ceq 20 30 15

So,
l'J. = .J~CI-X-2"""C::-I
= CI = .fi .J2 ~ 1 3+ 2+ 4 ~ _1_ = ~
V2 CI + 2CI 3 CI 3 (2) Ceq 60 Ceq 60
IT
60 20
30. When the capacitors are connected in parallel. ~ Ceq = - ~ Ceq = - JlF
. equivalent capacitance, Cp = CI + C2• 9 3~)
The energy stored in the combination of the Since, charge remains same in series
combination,
capacitors, Ep = ~CpV2 (112) 20
2 So, Q = CeqV ~ Q = - x 90
1 2 3
Ep = - (CI + C2) (100) = 0.25J
Q= 600 JlC~ Q = 600 X 10-6 C
H
2 ~
~ CI+C2=5XIO-5 ... (i) ~ Q = 6 X 10-4 C
When the capacitors are connected in series, .:.Potential difference across C2 = ~
equivalent capacitance, V2
Cs=~ ~ V - Q ~ V _ 6 X 10-4
O

CI + C2 2 - C2 2 - 30xl0-6
The energy stored in the combination of the
capacitors, ~ V2 = 0.2XI02 ~ V2 = 20V (1)
1 2 Also, energy stored in capacitor C2 is given by
s, =-CsV (1/2)
1 1
x io
2 2 2-6
E=-C2V2 ~E=-x30x(20)
M

2 2
s, = ~ ~ (100)2 = 0.45J
2 CI + C2 ~ E = ~ x 30 x 400 X 10-6 ~ E = 6000 X 10-6
2
~ X~ (100)2 = 0.45J
2 5xl0-5 ~ E = 6 X 10-3 J (1)

CIC2 = 0.045 x 10- 4 X 5 x 10- 5 X2 32. (i) The energy of a charged capacitor is measured
= 4.5 X 10-10
by the total work done in charging the
capacitor to a given potential.
{CI - C2)2 = {CI + C2)2 - 4CI C2 Let us assume that initially both the plates are
~ (CI - C2)2 = 25x 10-10 - 4 x 4.5x 10-10 uncharged. Now, we have to repeatedly
remove small positive charges from one plate
= 7xlO-10
and transfer them to other plate.
CHAPTER 2 : Electrostatic Potential And Capacitance 63

Now, when an additional small charge (dq) is Suppose, the plate X is given a charge of
transferred from one plate to another, the +q coulomb. By induction, -q coulomb of
small work done is given by charge is produced on the inner surface of the
dW = V'dq = f..dq plate Y and + q coulomb on the outer surface.
C (1/2) Since, the plate Y is connected to the earth, the
[let charge on plate, when dq charge is +q charge on the outer surface flows to the
transferred is q'] earth. Thus, the plates X and Y have equal and
The total work done in transferring charge Q is opposite charges.

R
given by Suppose, the surface density of charge on each
Q q' I iQ q'dq plate is a. We know that the intensity of
W= io -dq =-
ceo
electric field at a point between two plane
parallel sheets of equal and opposite charges is
a/Eo, where Eo is the permittivity of free space.
= ~ [(q')2]Q = Q2
The intensity of electric field between the

SI
C 2 0 2C (1/2)
plates will be given by, E = ~
This work is stored as electrostatic potential Eo
energy U in the capacitor. The charge on each plate is q and the area of
_ Q2 _ (CV)2 each plate is A. Thus,
U----- [':Q = CV]
2C 2C a =~ and E = _L ...(i)
A EoA
2
U = ~ CV
2 n) Now, let the potential difference between the
two plates be V volt. Then, the electric field
The energy stored per unit volume of space in a
IT
between the plates is given by
capacitor is called energy density.
E =~ or V = Ed
~CV2 d
u=_2 __ 1 eoAv2
~ U=---
Ad 2 d2A
Substituting the value of E from Eq. (i). we get
1 2 V=~
Energy density, U = - eo E EoA
2
H
Total energy stored in series combination or :.Capacitance of the capacitor is
parallel combination of capacitors is equal to C = ~ = -q-- or C = EoA
the sum of energies stored in individual V qd/EoA d
capacitor.
i.e. U=UI +U2+U3+ ... where, Eo = R,85xIO-12 C2_Nm-2
O

(ii) Due to conservative nature of electric force, It is clear from this formula that in order to
the work done in moving a charge in a close obtain high capacitance
path in a uniform electric field is zero. (1) (a) A should be large, i.e. the plates of large
area should be taken.
33. (i) Parallel plate capacitor consists of two thin
conducting plates each of area A held parallel to (b) d should be small, i.e. the plates should be
kept closer to each other. (2)
M

each other at a suitable distance d. One of the


plates is insulated and other is earthed. There (ii) Surface charge density is given by
is a vacuum between the plates. a = -q-
Surface X,( Area A
41tR2

Charge~
density .
1 I After connecting both the conductors,
potentials will become equal.
their

++++++++++ Vi = V2
lllEIII d ~ Kql = Kq2 [':for spherical conductors
~ ~
v= __
1
i or V=-]Kq
41tEo R R
Surface
charge ~
~
L
- - - - 2
y I
I ~~=Rl ~~=ql/41tR~ ql (R2)2_R2
density ~ Earthed
q2 R2 a2 q2 /41tR~ q2 RI RI (1)
64 o ehapterwise eSSE Salved Paper> PHYSICS

34. Given, area of each plate, A = 6 x 10-3 m2 (ii) Let Qbe the charge stored in the capacitor
Q = CV= 2x 10-6 x 120
Distance between plates d, = 3mm = 3 x 10-3 m
(i) Capacitance of paraliel plate capacitor is given Q = 24 X 1O-5C (112)
by
37. Given, C = 200 pF = 200 x 10-12 F and V = 300 V
C=EoA
d The energy (initially) stored by the capacitor is
U· = .!.CV2 = .!.x 200 X 10-12 X 300 x 300

R
Eo = 8.85XI0-12C2 N-lm-2
I 2 2
. 'llA[') I 885xlO-12X6xl0-3 = 9 X 10-6)
{I C =-.----~--
3xl0-3
The charge on the capacitor when charge through
I J I '1 C =;ol,;ZZ x 1O-11F (1)
300 V battery is

SI
(ii) Charge on parallel plate capacitor is given by
Q=CV
t-
Q=CV = 200 X 10-12 X 300
Given, V = 100V
Now, Q=17.7xl0-12 x 100 =6xlO-8C

Q=1.77XI0-lOC = 60 X 1O-9C = 60nC (1)


... (i) (1)
(ill) Given, K = 6 When two capacitors are connected, they have
Now, C'=KC their positive plates at the same potential and
.. O'=KQ [From Eq. (i)]
negative plates also at the same potential. Let V
IT
Q'=6 x 1.77 X10-10 be the common potential difference. By charge
(1)
conservation, charge would distribute but total
charge would remain constant.
35. According to Gauss's theorem, total flux ~ = .!L. Thus, Q = q+ q'
Eo
!!.. = q'
Applying the formula sphere of radius rl has
C c
H
electric flux ~1' and sphere of radius r2 has electric
flux ~2' then
...L=~
200 100
(i)~1 =~'~2=3Q ~=~
Eo Eo ~2 3 q = 2q'
(ii) if a medium of dielectric constant 5 is filled in Thus, Q = 2q' + q' = 3q'
O

the space inside SI< the flux inside SI So, q,=!l= 60nC=20nC
3 3
~'1 = ~ =~ [Replacing Q by ~1Eo]
5Eo 5 and q = 2q' = 40 nC [1)

Thus, final energy


36. (i) We have initial voltage, VI= V volt and charge
q2 q,2
stored, <4 = 360JlC
Uf = 2C + 2C'
M

Ql =CVJ ... (i)


Charged potential, V2= V -120 1 (40 X 10-9)2 1 (20 X 10-9)2
=-x +-x-'----~
Q2=120JlC (1/2) 2 200xl0 12 2 100xl0 12
Q2=CV2 ... (ii) = 4 X 10-6 + 2 X 10-6 = 6 X 10-6)
By dividing Eq. (ii) from Eq. (I), we get
. Ql _ CVJ 360 _ V Difference in energy
--- ~---- = final energy - initial energy
Q2 CV2 120 V -120
=Uf - U,
V=180V (1/2) = 6 X 10-6 - 9 X 10-6
-6
C = Ql = 360x1O = 2xlO-6 F = 2 JlF = - 3x 10-6
VI 180
Thus, difference in energy is - 3 X 10-6). (1)
Hence, the potential, V=180V and unknown
capacitance is 2 JlF. (Wo)
CHAPTER 2 : Electrostatic Potential And Capacitance 65

38. In a series combination, where there is no Energy stored later,


2
division of charge,
U,=-q- [·:C' = KC]
1 1 1 1 2/.KC)
-=-+-+-+ ...
C, C1 C2 C3 where, K = dielectric constant of medium
In a parallel combination, where potential
difference is same. ~ u,=~(ct) ~U'=~(U)~U'=~XU
Cp = C1 + C2 + C3··• K 2C K K
(i) Here, C1 ,C2 and C3 are in series, therefore, The energy stored in the capacitor decreases

R
their equivalent capacitance and becomes ~ times of original energy.
K (1)
1 1 1 1
-=-+-+-
C' C1 C2 C3 40. On introduction of dielectric slab to fill the gap
between plates of capacitor completely when
C'=~=~=4~F capacitor is connected with battery. ,j

SI
3 3 (1)
(i) The potential difference V between capacitors
Now, C' and C are in parallel combination. is same due to connectivity with battery and
hence, charge q' becomes K times of original
Cnot= C' + C
charge as
= 4 ~F+ 12~F= 16~F q' = C'V' = (KC) (V) = K(CV) = Kq
Cnot= 16 ~F (1) q' = Kq n)
(ii) Being C' and C are in parallel, 500 V potential (ii) Electric field intensity continue to be the same
difference is applied across them. as potential difference and separation between
two plates remain unaffected as
IT
:. Charge on C'
ql = C'V E=~.
d (1)
= (4 ~F)x 500 = 2000 ~C
(ill) The capacitance of capacitor becomes K times
:. C1 ,C2 and C3 capacitors each will have of original capacitor.
2000 ~Ccharge. .. C' = KC = K eo A
d
H
Charge on C4' q2 = C X V (1)

=12x500 41. After disconnection from battery and doubling


= 6000~C (1) the separation between two plates
39. On introduction of dielectric slab in an isolated (i) Charge on capacitor remains same.
charged capacitor. i.e. CV = C'V'
O

(i) The capacitance (C') becomes K times of


original capacitor as ~ CV = ( %) V' ~ V' = 2V
C = Eo A and C' = K eo A
d d n) Electric field between the plates
E'= V' = 2V
(ii) Charge remains conserved in this
d' 2d
M

phenomenon.
cv= C'V'
E'=..!::.=E
d
V' = cv = CV [refer part (i)]
C' KC ~ Electric field· between the two plates
remains same. (1)
V'=~ (ii) Capacitance reduces to half of original value as
K
Coo.!. ~ C'=~
Potential difference decreases and become ~ d 2 (1)
K
times of original value. (1) (ill) Energy stored in the capacitor before
(ill) Energy stored initially, disconnection from battery
2 q2
U=!L U1 =-
2C 2C
66 o Chapterwise CBSE Solved Papers PHYSICS

Now, energy stored in the capacitor after 44. (i) The total charge on the capacitor remains
disconnection from battery conserved on introduction of dielectric slab. Also,
2 2 2 the capacitance of capacitor increases to K times
U-q - q -q
2- 2(C') --(C)-C of original values.
2x -
2
.. CV = C'V'
CV = (KG) V' => V' = ..!:::.
=> U2 = 2(~)= 2UI U2 = 2UI K
:. New electric field,

R
Energy stored in capacitor gets doubled to its
(tJ initial value. (1)
E'= :' =(V~K)=(~)~=f

42. As we know, to determine the electric field at any :. On introduction of dielectric medium, new
point at distance r from centre if we apply electric field E' becomes ~ times of its original
Ga~ss' tth~orem K

SI
value. (1)
E = q
(ii) .: Capacitance of a parallel plate capacitor
41tEo . r2
partially filled with dielectric medium is given
Hence, V = f E· dr = --q-
41tEo . r
by
C= EoA
where.e, = 8.854 x 10-12 C2 N-I m " (d - t + t/K) (1)

The capacitance of the spherical conductor where, t is the thickness of dielectric medium.
situated in vacuum is given by Here, t=~
2
C=!!...=--q-
IT
V _1_.:i C= EoA
41tEo r d-~+~
2 2K
=> C = 41tEor.
Hence, the capacitance of an isolated spherical C= 2EoAK
conductor situated in vacuum is 41tEo times of its (K + 1) d (1)
radius. 45. (i)
H
The graph comparing the variation of
43. Let VI and V2 are the potential differences across potential V and electric field. Refer to Ans. 26,
the series and parallel combination of two Topic -1. (1)
identical capacitors each of capacitance C. (ii)
Let CI = C and C2 = 2C
Equivalent capacitance in series combination
O

C =!:.. :. Equivalent capacitance


, 2 n) In series, C = 2C x C = 2C = 2C
2

, 2C+C 3C 3
Equivalent capacitance in parallel combination
c, = 2C In parallel, Cp = 2C + C = 3C (1)

According to the question, . . Vp and V, are potential difference across the


M

U, = Up (1) final capacitor in parallel and series combination


respectively, to have same potential energy.
~ C V2 = ~ C V2
2 s 2 p p Up = U,
V/ _ Cp _ 2C ~ PPC V2 = ~ C V2
=>
,") ) V -C,-(%)
p
2 2
Vp _
2"
K
2 v, V Cp
V,=4=>V,=2
Vp2 Vp 11= /2C/3) = ~
V,:Vp=2:1 (1)
(3C) V"9
Vp:V,=.fi:3 (1)
CHAPTER 2 : Electrostatic Potential And Capacitance 67

46. In series combination, charge on each capacitor is (iii) Energy stored in the network of capacitors
the same. q,2 q,2 q2
U = U1 + U2 + U3 = - + - + -
(i) All capacitors of 61lF are in series combination, 2C1 2C2 2C3
then equivalent capacitance is given by
1 1 1 1 ': C1 = C2 = C3 = 61lF
-=-+-+-
C' C1 C2 C3 .. U = _1_ [q,2 + q" + q2]
(l21lF)
or C' = ~ = 6 IlF = 21lF
n 3 = _1 _ [(361lq2 + (361lq2 + (nllC)2]

R
(l21lF) t.J
I,

C' and 21lFcapacitors are in parallel combination.


U = 6481l) .... ilIJ· (1)
:. Equivalent capacitance
49. According to the question,
Ceq= C' + 21lF Capacitors of 20 IlF and C are connected in series.
= 21lF + 21lF (i) The equivalent capacitance

SI
,J J

Ceq= 41lF (1)


(4 IIF) = (20 IlF) x C
(ii) Since, C' and 2 IlFare in parallel combination, r- 20+ C
therefore, same potential difference 6 V is
(20 + C) = 5C 4C = 20
applied on them.
C = 51lF (1)
:. Charge on C' (ii) Charge on capacitor (equivalent)
q' = C'V = (21lF) x 6 V = 121lC q = (4IlF) x12= 481lC
same charge 481lClies on both the capacitors. (1)
IT
The charge across the each capacitor of 61lF (ill) Potential drop across 20 IlF capacitor
capacitor is same and equal to charge across the
Vi =..i. = 481lC = 2.4 V
combination i.e. 121lC
C1 20llF

Charge on 21lF capacitor q = CV = (21lF) (6 V) Potential drop across 5 IlF capacitor


= 121lC (112 x 4 = 2) V = ..i. = 481lC = 9.6 V
2
C2 5j.1F
H
47. (i) Refer to Ans. 44 (i). (2)

(ii) Refer to Ans. 39 (iii) (1)


50. A dielectric medium is inserted between the
platesof a condenser in place of air, its capacity
48. (i) The equivalent capacitance of C1 and C2 becomesx times of original one.
connected in series
The capacitance of two capacitors are
1 1 1
O

-=-+-
C' C1 C2 C = Eo A =C [say]
x d
C'=~=3IlF C = 4EoA =4C
2 y d
:. Charge, q' = C'V = (3IlF) 12 = 361lC
(i) According to the problem,
:. Charge on each capadtor of C1 and C2 is 361lC
M

Ceq= 41lF
Charge on C3' c, xCy
q3 = C3V = (6IlF) x 12= nllc (1) .. Ceq=---
c, + Cy
q3 = 72IlC
4IlF=CX4C=4C
(ii) Equivalent capacitance of network, C+ 4C 5
.: For series combination, C= ~ lJ
Ceq=~+ C3 [ C1 + C2
C1 + C2
C = 51lF
=6X6+6
6+ 6 => Cx=C=5IlF
Cy = 4C = 4 x 51lF
=3+6=9IlF
= 20 IlF (1/2 x 2)
Ceq= 91lF (1)
68 (2] Chapterwise CBSE Solved Papers PHYSICS

(ii) Charge across the combination ~+Z=8 =>Z=8-~ _32-15


q = Ceq V 4 4 4
17
= (4~F) x 12 Z = - ~F => Z = 4.25 ~F
4 m
=48~C
52. (i) According to question
.. Same charge, i.e. 48 ~C lies on each capacitor
being in series combination. +
+
.. Potential difference across Cx'
+ E
Vi =.!!.... = 48 ~C = 9.6 V + ~- •

R
c, 5~F + P Q
Potential difference across Cy' +
V =.!!.... = 48 ~C = 2.4 V A + - A
2 I+-- d--+l
Cy 20 ~F (1/2 x 2)
(a) Electric field due to a plate of positive

SI
(ill) Energy stored in capacitance X-
2 charge at point P = ~
U =-q- 2Eo
x 2Cx
Electric field due to other plate = ~
2Eo
'~y stored in capacitor Y, Since, they have same direction, so
2
U =!L E =~+~=~
y 2Cy 2Eo 2Eo Eo

[ Since, in series, the charge will remain same] Outside the plate, electric field be zero
IT
. Ux = ~ x 2C2y = Cy = 20 ~F = 4 because of opposite direction. (1)
• • (b) Potential difference between the plates is
Uy 2Cx q c, 5~F
given by
Ux:Uy=4:1 (1)
V _. Ed= od
51. Given, total energy =160mJ Eo (1)
(c) Capacitance of the capacitor is given by
H
Let equivalent capacitance of the combination of
capacitors is C. (':Q = CV)

According to the question, C= g


= aA Eo = EoA
V ed d (1)
~ CV 2
=160X10-3 J (ii) According to question,
2
O

=> ~ x C X (200)2 = 160 X 10-3


2
=> Equivalent capacitance, C = 8 ~F (1)

.: Equivalent capacitor of 7 ~F and 3 ~F connected


in parallel = 7 + 3 = 10 ~F Potential at the surface of radius R,
M

:. 1O~F,1O~Fcombinationof7~F, 3~F and l suf = kq [':q = 0 x 4R2]


are in series combination. Therefore, their R
ko4rcR2
equivalent capacitance => --- = ok4rcR = 4kCJ1tR
1 1 1 1 3+ 3+ 2 8 R
-=-+-+'- Potential at the surface of radius 2R,
C' 10 10 15 30 30
C' _15
= kq [':q = 0 x 4rc(2R)2 = 16CJ1tR2]
-4~ F (1) 2
2R

So, ko16rcR = 8kCJ1tR


But, C' and Z are in parallel combination, 2R
therefore equivalent capacitance Since, the potential of bigger sphere is more. So,
~ + Z=C [from Eq.(i)] charge will flow from sphere of radius 2R to sphere
4 of radius R. (2)
CHAPTER 2 Electrostatic Potential And Capacitance 69

53. (i) When a capacitor is placed in an external till conductor 1 get charge +Q. By charge
electric field, the free charges present inside conservation, conductor 2 would get charge
the conductor redistribute themselves in such -Q.
a manner that the electric field due to induced
charges opposes the external field within the
conductor. This happens until a static
+Qt==J-Q+-
situation is achieved, i.e. when the two fields +-

R
cancels each other and the net electrostatic +-
field in the conductor becomes zero. +-
+-
In contrast to conductors, dielectrics are 1 2
non-conducting substances. i.e. they have no At every stage of charging, conductor 1 is at
charge carriers. Thus, in a dielectric, free higher potential than conductor 2. Therefore,

SI
movement of charges is not possible. It turns work is done externally in transferring each
out that the external field induces dipole installment of charge.
moment by stretching molecules of the :. Potential difference between conductors 1 and
dielectric. The collective effect of all the
2 isi
molecular dipole moments is the net charge C
on the surface of the dielectric which
produces a field that opposes the external .. Potential of condenser = i .
C
field. However, the opposing field is so
induced that does not exactly cancel the Small amount of work done in giving an
IT
external field. It only reduces it. The extent of additional charge dqto the condenser,
the effect depends on the nature of dielectric. dW=ixdq
Both polar and non-polar dielectric develop C
net dipole moment in the presence of an :. Total work done in giving a charge Q to the
external field. The dipole moment per unit condenser,
volume is called polarisation and is denoted
q=Qq 1 [q21q=Q
H
by P for linear isotropic dielectrics. (2'12)
W= j q=OC=C 2'1
P=XE -1,=0
(ii) (a) At point C, inside the shell. 1 Q2
The electric field inside a spherical shell is ~ W=--
C 2
zero. Thus, the force experienced by
charge at the centre C will also be zero. As, electrostatic force is conservative, this work is
O

stored in the form of potential energy (U) of the


.: Fe = qE (Einside the shell = 0)
:. Fe = 0
condenser.
Id
At point A, IFAI = 2Q[_1_ 3;]
41tEo X2
U=W=--
2C
Q=CV
M

3Q2 U = ~ (CV)2 = ~CV2


F = ---2' away from shell 2 C 2
47tEox
1
(b) Electric flux through the shell, CV=Q ~ U=-QV
2
~ = ~ x magnitude of the charge enclosed
Eo 1d 1 2 1
Hence, U=-- =-CV =-QV
by the shell. 2 C 2 2
~ ~=~xg=~ Energy density (U) is defined as the total energy
Eo 2 2 Eo (2'12) per unit volume of the condenser.
54. (i) In order to calculate the energy stored in the ~CV2
charge configuration, suppose the conductors i.e. U = Total energy (V) =_2_
1 and 2 are initially uncharge. Let positive Volume (V) Ad
charge be transferred from conductor 2 to
Using C = EoA and V = Ed
conductor 1 in very small installments of each d
70 o ehapterwise eSSE Solved Papers PHYSICS

We get, U="21 (EoA)


d (EAd 2
2d ) 1
="2EoE
2
V = Vi + V = _1_.
2
41tEo
(-q)
r + a
+ _1_._q_
41tEo r - a

Here, E is the strength of electric field in the = _q_[_I


41tEo r- a r+a
1_] = .s.i.:»:
41tEo r2 - a2
space between the plates of the capacitor. (21/2)

(ii) Initial condition 1 P


[':p = q (2a)]
= 41tEo . r2 _ a2
If we consider a charge capacitor, then its charge
would be given, q = CV For a far away point, r » > a

R
«c» .. V = _1_.!... or V oc ~
41tEo r2 r2
A"'--H--B
Thus, due to a dipole potential at a point is V oc -; .
and energy stored in it is given by r

SI
(2112)
1
UI = -cv 2
...{i) (ii) Let A -7 area of each plate and CI and C2 are
2
capacitance of each slab.
When this charged capacitor is connected to
...
L et iruna 11y, CI = C = -EoA = (,2,
uncharged capacitor,
d

e~
C1·V1 After inserting respective dielectric slabs.
C; =KC
' - K 1---+
an d C2- Eo{A/2) K Eo{A/2) - EoA (K 1+K 2').
2-----
d d 2d
IT
C2' = ~ (KI + K,) ... (ii)
Let the common potential be Vi, the charge flow 2 -
from first capacitor to the other capacitor unless From Eqs. (i) and (ii). we get
both the capacitor attain the common potential. "C 1
CI =C2~KC=-{KI +K2)~K=-{KI +K2)
~ QI = CVi and Q2 = CV2 2 2 (2112)

56. (i) For (a) and (c), refer to Ans. 39{i) and (ill)
H
Applying conservation of charge,
Q=QI + Q2 ~ CV=CVI + CV2 For (b), refer to Ans. 44 (i). (3)
(ii) (a) Electric field lines due to a conducting
V
~ V = Vi + v, ~ VI =- sphere are shown in the figure.
- 2
Total energy stored on both the capacitor
O

I
U2 = -CV I
2 + -CV
1
I
2 ~ 1 ( -V
U2 = -C )2 1 ( -V )2
+ -C ---"-{V)----
2 2 2 2 2 2
- - - ~ Conducting
2CV2 I 2 sphere having
U2 = -- = -CV ... {ii)
8 4 negative charge (1)
M

From Eqs. (i) and (ii). we get (b) Electric field lines due to an electric dipole
U2< UI are shown in the figure
It means that energy stored in the combination is p -E
less than that stored initially in the single
capacitor. (2112)

55. (i)
-qo I o---------- .• p
o +q
~------ r ------~
Let P be an axial point at distance r from the centre
of the dipole. Electric potential at point P will be
(1)
CHAPTER 2 : Electrostatic Potential And Capacitance 71

57. As, charge on capacitor increases, we have to 58. (i) Refer to Ans. 39 (i). (3)
work more against electrostatic repulsion and this (ii) Refer to Ans. 32 (i). (2)
amount of work done will be stored as potential
59. Let small sphere has charge q and radius a is
energy in the capacitor.
placed inside a outer shell of charge + Q and
(i) We know that, q =CV ~ V =qt C
radius b.
dW = Vdq = !idq
C Insulated
___~_suspension
where, q = instantaneous charge,
C = instantaneous capacitance and

R
V = instantaneous voltage +0
:. Total work done in storing charge from 0 to q
q 2
f
is given by W = !idq =!L
oC 2C (2)

SI
(ii) In series combination of capacitors, same
charge lie on each capacitor for any value of
Electric potential on the small sphere due to its
capacitances.
own charge q.
+0 -0 +0 -Q +O-Q

!=H!=H!=
1 q
Vi =--'- ...(i)
41tEo a
+- +- +-
+- +- +- where, q = charge on small sphere
C1 C2 C3 a = radius of small sphere
V1 ------1- V2 --+- V3 Similarly, electric potential on outer sphere due to
IT
its own charge
v V2=_I_.Q ... (ii)
(+) (-)
41tEo b
Capacitors in series combination (1)
where, Q = charge of outer shell
Also, potential difference across the combination b = radius of outer shell. (1 x 2 = 2)
is equal to the algebraic sum of potential
H
differences across each capacitor. Also, same potential V2 exists at every point inside
outer shell due to its own charge, + Q.
i.e. V=Vi+V2+V} ... (i)
Now, net electric potential at inner sphere of
where, Vi ' V2, V} and V are the potential
radius a.
differences across C, ' C2, C} and equivalent
capacitor, respectively. 1'; = Electric potential due to its own charge
1 q 1 Q
O

Vi =~ 1'; = -- - + --- ... (iii)


C, 41tEo a 41tEo b

Net electric potential at outer sphere due to charge


Similarly, V2 = ~ ~ V} = ~
C2 C} on the both spheres
:. Total potential difference [from Eq. (i)] 1 q 1 Q
Vo =--'- + --'- ... (iv)
M

41tEo b 41tEo b
V=~+~+~
C, C2 C} (1) .. 1'; - Vo = -q- (2.. - 2..) ... (v)
VIII 1111 41tEo a b
-=-+-+-~-=-+-+-
q C, C2 C} C C, C2 C} From Eqs. (iii) and (iv), we get
1 1
[.: ~ = 2.., where C is equivalent capacitance of ': a < b and - > - ~ 1'; - Vo > 0
a b (2)
q C
combination] Thus, inner sphere has net potential higher than
or 2.. = C2 C} + C} C, + C, C2 potential of outer sphere for every value of q and Q.
C C, C2 C} Therefore, when they are connected by a wire,
positive charge always flow from inner sphere (at
C= C, C2 C} higher potential) to outer sphere (at lower potential)
C, C2 + C2 C} + C} C, (1) irrespective of the magnitude of charge. (1)
Value Based Questions (From Complete Chapter)
rlI 4 Marks Questions Ans. (i) Electricalenergy is stored in the capacitor. It is
stored in the dielectric. [1]
1. Immediatly after school hour, as Bimla (ii) No effect. When the metal sheet is placed in
with her friends came out, they noticed the middle the new arrangement is equivalent
that there was a sudden thunderstorm to a old combination of two capacitors each of
accompanied by the lightning. They could plate separation t!.- and hence capacitance 2C.

R
not find any suitable place for shelter. Dr. 2
Kapoor who was passing thereby in his C = 2Cx2C =C
s
car noticed these children and offered 2C+ 2C (1)

them to come in their car. He even took (iii) Team work, concern, respect to teacher and
care to drop them to the locality where responsibility. [2]

SI
they were staying. Bimla's parents, who
3. An old woman who had suffered from a
were waiting, saw this and expressed
'heat stroke was taken to the hospital by
their gratitude to Dr. Kapoor. All Indio 2015 C
her grandson who is in class XII. The
(i) What values did Dr. Kapoor and grandson has studied in Physics that, how
Bimla's parent displayed?
to save person who is suffering from a
(ii) Why is it considered safe to be inside a heat stroke, regular beating of the heat is
car especially during lightening and
to be restored by delivering a jolt to the
thunderstorm?
IT
heart using a defibrillator, whose capacity
(iii) Define the term 'dielectric strength'.
is 70 IlF and charged to a potential of
What does this term signify?
5000 V and energy stored is 875 J, 200 J
••••• (i) Helping attitude, kindness, concern for the of energy is passed through a person's
children, awareness, application of knowledge.
(1)
body in a pulse lasting 2 ms. The old
woman gets paniced and refuses to be
(ii) As car is a good conductor, its made up of
H
metallic body so it will behave as a lightning treated by defibrillator. Her grandson
conductor. (1) then explained to her the process that
(iii) The maximum electic field that can exist in a would be adopted by medical staff and
dielectric without causing the breakdown of how the result of that would bring her
its insulating property is called dielectric back to normalcy. The woman was then
O

strength. It is usually expressed as volts/unit treated and was back to normal.


thickness. (2)
Answer the following questions based on
2. In Pradeep's classroom, the fan was the above information.
running very slowly. Due to which, his (i) What according to you are the values
teacher was sweating and was restless displayed by the grandson?
M

and tired. All his classmates wanted to


(ii) What will be the net charge of the
rectify this. They called an electrician who
capacitor in defibrillator?
came and changed the capacitor only,
after which the fan started running fast. Ans. (i)Presence of mind, knowledge of subject,
concern for his grandmother, empathy helping
Answer the following questions based on and caring, [2]
the above information. (ii) The net charge of the capacitor in defibrillator
(i) What energy is stored in the capacitor is given by
and where? where, C = capacitance]
Q=CV
(ii) A thin metal sheet is placed in the [ V = voltage
middle of a parallel plate capacitor. What
Q = 70 X 1O-{i X 5000 = 35 X 10-2C [2]
will be the effect on the capacitance?
(iii) What values did the classmates have?
CHAPTER 2 : Electrostatic Potential And Capacitance 73

4. Shikhaj was working on a project for 5. A man travelling in a car during heavy rain
science exhibition. He considered a and thunderstorm, sees a boy standing .,
capacitance of 2 ~F having a capacity of under a tree. He immediately stops his car
operating under 1 kV potential. When and asks the boy to get inside the car and,'il
he reached to shop, he found that the saves him from a possible natural calamity,
shopkeeper is having a capacitors of Answer the following questions based on the
1 ~F of 400 V rating. Shikhaj calculated above information.

R
minimum number of capacitances of (i) What danger did the boy had while
1 ~F each, so he could arrange to form a standing under the tree during the
capacitor of 2 ~F. th understorm?
Answer the following questions based (ii) How is it safer inside the car during
on the above information. such weather?

SI
(i) What are the calculations done by (iii) What according to you are the values
Shikhaj? displayed by the man to help the boy?
(ii) What do you think of Shikhaj? (iv) Give another example from everyday
Ans. (i) Total potential difference across each row life situations which represent display
=1000 V of similar values.
Potential difference across each capacitor Ans. (i) Trees often being much taller than surrounding
=400V structures, will attract lightning and the tree can
get on fire. The boy might get electrocuted. Also,
IT
:. Number of capacitors in series,
if the lightning strikes the tree, then the tree
n = 1000 = 2.5 = 3 might fall on the boy. (1)
400
(ii) During thunderstorms, it is safer to -sit inside a
Capacitance of capacitors in series, C, =!. IlF car because car has a metallic body that does not
3 allow the lightning to hit the person sitting· ",,",
(1) inside the car. We know that charges always
H
Let m be rows of capacitors, for an reside on the surface of metal. Inside the
equivalent capacitance of 21lF, we have conductor, charge is zero. If lightning strikes the
mx!'=2 car during a thunderstorm, the charges will be
3 distributed on the surface of the metal body of
~ m=6 car. No charges will exist inside the car. Hence,
O

the person sitting inside the car is


Hence, total number of capacitance required
considered safe are the values displayed. (1)
= mx n
(ill) Helpfulness; Compassion, General awareness,
=6x3 service to others are the values displayed. (1)
=18 (iv) Advising a person who is doing repair work on
Total 6 rows of capacitor in parallel with air condition system without switching OFF the
M

three capacitors in each row. (1) mains.


(ii) We think that Shikhaj knows the value of Stopping a person entering nuclear medicine
money and have the ability to calculate ward in a hospital without proper protection or
scientifically in daily life. (2)
precautions. [1/2 + 112)

You might also like